Archer 9

Pataasin ang iyong marka sa homework at exams ngayon gamit ang Quizwiz!

D

A 14-year-old is admitted to the medical ward for status asthmaticus. He was put on IV theophylline. Which manifestation would the nurse consider as a side effect of the drug? A. Grand mal seizures B. Severe palpitations C. Hypotension D. Headache

C, D

Which of the following signs and symptoms indicate right-sided heart failure in a pediatric patient? Select all that apply. A. Grunting B. Nasal flaring C. Ascites D. Hepatosplenomegaly

D Choice D is correct. The collection of vital signs may be delegated to a UAP. This includes pulse, blood pressure, temperature, and oxygen saturation. Choices A, B, and C are incorrect. Oxygen is regarded as medication, and any adjustment or application of oxygen is not within the scope of a UAP. Venipuncture is not within the scope of a UAP, and it would be inappropriate for this task to be delegated. Removing a vascular access device may be delegated to an LPN - not a UAP.

The nurse is caring for a newly admitted client. Which task can the nurse delegate to the unlicensed assistive personnel (UAP)? A. Apply nasal cannula oxygen B. Remove a vascular access device that is not patent C. Perform venipuncture for laboratory work D. Obtain vital signs every four hours

B

Your client has been diagnosed with acute renal failure. Which one of the following lab results should be reported immediately? A. Blood urea nitrogen 50 mg/dL B. Serum potassium 6 mEq/L C. Venous blood pH 7.30 D. Hemoglobin of 10.3 mg/dL

B

The nurse is assigned the case manager role. She understands that case management uses which of the following methods of patient care delivery and documentation? A. A problem-oriented documentation system. B. A critical pathway documentation system. C. A source-oriented documentation system. D. A variance-oriented documentation system.

A, D

The nurse is assisting with monitoring a client that has a chest tube and documents the appropriate assessments. Which of these assessments are expected findings? Select all that apply. A. Drainage system at a level below the patient's chest. B. Vigorous bubbling in the water-seal chamber. C. Stable water in the tube of the water-seal chamber during inhalation and exhalation. D. Occlusive dressing over the chest tube.

C

The nurse is attending to a client who is 20 weeks pregnant and has completed patient education. Which of the following statements by the client indicates that she has a good understanding of her baby's development? A. "My baby is able to breathe now." B. "My baby can open his eyes." C. "My baby is about 7 ½ inches long." D. "My baby has fully grown fingernails."

C

The nurse is caring for a 7-year-old child who is continuously anxious in the pediatric ward. The nurse plans to initiate therapeutic play to help the child's anxiety. During the therapeutic game, all of the following activities should be included, except: A. Encourage manipulation of equipment. B. Constantly monitor the child's anxiety levels throughout the activity. C. Provide structure for the play. D. Continue play sessions regularly.

A

The nurse is caring for a client immediately following the administration of prescribed sumatriptan. Which clinical finding would require follow-up by the nurse? A. Blood Pressure 189/98 mm Hg B. Headache pain 4/10 C. Client's skin appears flushed D. Reports of nausea

C

The nurse is caring for a client with a potassium of 3.2 mEq/dl. Which of the following medications may cause this abnormality? A. Spironolactone B. Triamterene C. Prednisone D. Lisinopril

A

The nurse is caring for assigned clients. Based on the pulse (P), respiratory rate (R), and blood pressure (BP) provided, it would be essential to follow up with which of the following clients? A. P: 109; R: 26; BP: 110/70 mmHg B. P: 90; R: 12; BP: 99/54 mmHg C. P: 100; R: 18; BP: 161/98 mmHg D. P: 88; R: 14; BP: 166/52 mmHg

B, E

The nurse is caring for assigned clients. Which of the following clients would be appropriate for the nurse to refer for an interdisciplinary conference? A client with Select all that apply. A. pulmonary tuberculosis with multiple prescriptions. B. ischemic stroke who has left-sided hemiplegia. C. hyperthyroidism and is scheduled for a thyroidectomy. D. stage one Alzheimer's disease who lives with family. E. fractured tibia and fibula and is homeless. F. end-stage-renal disease who refuses dialysis.

Apply the gown Secure the mask Apply the goggles/face shield Don gloves

The nurse is instructing a nursing student on the correct application of personal protective equipment (PPE). The nurse should tell the student to apply the PPE in what order? Place each action in the correct order.

D

The nurse is performing a head-to-toe assessment of the patient. During the abdominal evaluation, the correct sequence for this assessment is: A. Auscultation, Inspection, Palpation, Percussion B. Inspection, Palpation, Auscultation, Percussion C. Percussion, Auscultation, Palpation, Inspection D. Inspection, Auscultation, Percussion, Palpation

A, D

The nurse is planning a staff development conference about measures to reduce medication errors. It would be appropriate for the nurse to state which actions may help reduce medication errors? Select all that apply. A. Timely medication reconciliation B. Delay documentation of medication administration to the end of the shift C. Delegate medication transcription to unlicensed assistive personnel (UAP) D. Limit the use of verbal orders to emergent situations E. Place medication dispensing systems in high-traffic areas

C

The nurse is preparing to administer ear drops to a client who is six years old. The nurse should perform which action? A. Pull the ear pinna down and back B. Position the client on their side with the ear to be treated against a pillow C. Pull the ear pinna up and back D. Place cotton directly into the ear canal after ear drop administration

B, C

The nurse is preparing to administer prescribed medications to a client via a nasogastric tube connected to low-intermittent suction. The nurse should take which appropriate action? Select all that apply. A. Position the patient in Trendelenburg position. B. Verify correct placement of the tube before medication administration. C. Turn off the suction during medication administration. D. Resume low-intermittent wall suction immediately after medication administration E. Irrigate the nasogastric tube (NGT) with sterile water.

A

The nurse is preparing to give an intramuscular (IM) injection into the client's vastus lateralis. It would indicate the correct technique if the nurse A. palpates to find greater trochanter and knee joints; divide the vertical distance between these two landmarks into thirds; inject into the middle third. B. locates acromion process; inject only into the upper third of muscle that begins about two fingerbreadths below the acromion. C. locates the greater trochanter, iliac tubercle, and iliac crest; places palm over the greater trochanter, over iliac tubercle, along the ileum; inject into center of V formed by the fingers. D. displaces the skin by pulling the skin down or to one side about 1 inch with the nondominant hand before administering the injection.

A

The primary healthcare provider (PHCP) plans for a chest tube to be discontinued. Prior to the PHCP removing the chest tube, the nurse should take which action? A. Administer prescribed pain medication B. Apply nasal cannula oxygen at 4 liters per minute C. Clamp the chest tube two hours before the removal D. Instruct the client to empty their bladder

167 mL/hr

The primary healthcare provider (PHCP) prescribes one liter of 0.9% saline to infuse over 6 hours. How many mL per hour will be administered to the client? Fill in the blank. Round your answer to the nearest whole number.

A, B

The registered nurse is asked to assist the physician with removal of a chest tube. Which steps does the nurse anticipate will occur during the procedure? Select all that apply. A. Placing an occlusive dressing over the site. B. Asking the client to bear down as the tube is removed. C. Clamping the chest tube for 30 minutes prior to removal. D. Placing the drainage system near the head of the bed.

A

You are caring for a 1-month-old patient who has a sudden cardiac arrest. Which pulse should you palpate to determine circulatory status? A. Brachial B. Femoral C. Carotid D. Popliteal

C, D

You are providing education to an HIV+ mother about what she will need to do for her baby after he is born. Which of the following teaching points are appropriate? Select all that apply. A. Follow exclusive breastfeeding to ensure your infant receives proper nutrition. B. Ensure your infant receives all vaccines on time. C. Monitor your infant closely for signs of HIV for at least 18 months. D. Keep track of your baby's weight gain and notify the pediatrician if he doesn't gain weight as expected.

A, B, D

The nurse is assessing a client with Lyme disease. Which of the following would be an expected finding? Select all that apply. A. Lymphadenopathy B. Fatigue C. Petechial rash D. Arthralgias E. Hemoptysis

20 mL/hr

The nurse is caring for a client prescribed IV heparin. The client is prescribed 18 units/kg/hr. The client weighs 246 pounds. The heparin is labeled with 25,000 units in 250 mL of D5W. How many mL/hr should this client receive? Round your answer to the nearest whole number. Fill in the blank.

B

The nurse is caring for a client receiving mechanical ventilation. Which of the following actions would not reduce the client's risk of ventilator-acquired pneumonia (VAP)? A. Oral care with chlorhexidine B. Obtain a specimen for culture via tracheal suctioning C. Elevate the head of the bed to 45 degrees D. Performing hand hygiene before and after suctioning

A, B, C

The nurse is caring for a client who was prescribed carbidopa and levodopa for Parkinson's disease. The nurse should instruct the client that this medication may cause Select all that apply A. urine to appear darker. B. hallucinations. C. dizziness upon standing. D. dry, non-productive cough. E. painful rash that spreads and blisters.

A

The nurse is caring for a patient with left-sided heart failure. Which of the following signs and symptoms is related more to right-sided heart failure? A. Ascites B. Tachypnea C. Cough D. Crackles and wheezes

A, B, C

The nurse is caring for a terminally ill 8-year-old boy with leukemia. The health care provider expects he will only live for another 2-3 months. The nurse should know which of the following when explaining to him about the concept of death and illness? Select all that apply. A. School-age children do not understand their own mortality. B. School-age children feel very vulnerable when dealing with an illness. C. Each child is unique in how they process their diagnosis. D. The child will express their desire to tie up loose ends.

A, E

The nurse is caring for assigned clients. The nurse should recognize which client is at risk of developing hypoglycemia? Select all that apply. A client A. with diabetic ketoacidosis receiving continuous regular insulin intravenously. B. receiving methylprednisolone for an exacerbation of asthma. C. with pancreatitis and is receiving total parenteral nutrition (TPN). D. who is nothing by mouth (NPO) status following a coronary artery bypass graft (CABG). E. who received six units of lispro insulin one hour ago and has not eaten.

A, B, D, E

The nurse is discharging an adolescent with sickle cell disease. Which statement should the nurse include in the teaching? Select all that apply. A. Keep a water bottle with you at school so that you can stay hydrated B. Follow a high-calorie, high-protein diet C. Do not take the annual influenza vaccine D. Drink extra fluids if you have to travel on an airplane E. Daily aerobic exercise is recommended

B

The nurse is preparing to insert a nasogastric tube (NGT). Which action should the nurse take? A. Rinse the tube with warm, soapy water B. Perform hand hygiene C. Don sterile gloves D. Obtain a computed tomography (CT) scan to verify placement.

B

The patient that has just undergone cardiac surgery is recovering in the post-anesthetic care unit. The nurse notices that the patient's blood pressure is 88/52 mmHg and that his jugular veins are very prominent. The nurse auscultates his heart rate and cannot hear any heart sounds. The nurse immediately informs the physician on duty and prepares for which procedure: A. Thoracentesis B. Pericardiocentesis C. Arthrocentesis D. Paracentesis

A, D

Which of the following would cause an increase in cardiac output? Select all that apply. A. 2 L normal saline fluid bolus B. Furosemide C. Propranolol D. Dopamine

D

A nurse is assigned to care for four clients who are each one day postpartum. Following the nurse performing an initial assessment on each client, which finding would prompt further evaluation by the nurse? A. A client complaining of mild pain B. A client with a pulse rate of 65 bpm C. A client with colostrum discharge from both breasts D. A client with red, foul-smelling lochia

D

A client is being discharged following the insertion of a permanent pacemaker. Which of the following should be included in the client's discharge instructions? A. Air travel will not be possible due to airport screening equipment. B. You will need to discard any radios at home that have antennas. C. Computed tomography (CT) scans are not permitted with this device. D. You should use your cellular phone on the opposite side of the generator.

A

The nurse is caring for a client at risk of developing tumor lysis syndrome (TLS). Which of the following prescriptions would prevent TLS? A. Intravenous (IV) hydration B. Broad-spectrum antibiotics C. High dose corticosteroids D. Histamine-2 receptor antagonists

A

The nurse is caring for a client experiencing digitalis toxicity. The nurse anticipates a prescription for which medication? A. Digoxin immune fab B. Milrinone C. Amrinone D. Flecainide

A, C

Which of the following are true regarding physiological changes during pregnancy? Select all that apply. A. Increase in heart size B. Increase in gastric motility C. Reduced renal threshold for glucose D. Decreased basal metabolic rate

D

The patient is diagnosed with atrial fibrillation. What assessment data would require immediate intervention by the RN? A. Irregular QRS complexes on telemetry reading B. Rapid, irregular pulse C. The patient reports palpitations D. The patient reports lightheadedness

B, C

You are monitoring an 18-month old patient who has just had surgical correction of an epispadias completed. Which of the following assessment findings would the nurse need to report to the healthcare provider? Select all that apply. A. Urine output of 2.2 mL/Kg/hr. B. Cloudy, foul smelling urine. C. Stent in the meatus appears clogged. D. Temperature of 37.4 degrees Celsius.

D

A depressed pregnant patient is being seen in the clinic. Her physician has suggested that she try an anti-depressant to treat the condition. But the patient is nervous. The nurse should explain that all of the following are possible outcomes of untreated depression, except: A. Teratogenicity B. Non-adherence to prenatal care C. Tobacco use D. Respiratory distress post-birth

D

A nurse is preparing the plan of care for a client with stage 2 ovarian cancer who is a Jehovah's Witness. The client has been told that surgery is necessary. After discussing the client's religious preferences with the client, the nurse documents which of the following while creating the care plan? A. Religious sacraments and traditions are unimportant B. Medication administration is not allowed for this client C. Surgery is strictly prohibited for this client D. Administration of blood or blood products is not allowed for this client based on religious beliefs

A, B, D Choices A, B, and D are correct. After splenectomy, the patient is at high risk for developing OPSI (overwhelming post-splenectomy infection) and the nurse should recognize signs of an infection early on. Administering antibiotics and antipyretics for a fever is crucial to prevent the disease from worsening. The doctor should be called immediately because further treatment may be necessary. Choices C and E are incorrect. Palpating the patient's RUQ will not tell the nurse any vital information since the spleen would be palpated on the LUQ. The patient should not be placed in the Trendelenburg position because this will have no change in the patient's status.

A nurse on the surgical floor is caring for a patient who is three days post-splenectomy. During 0700 vital signs, the CNA obtains a 100.2-degree temperature but forgets to tell the nurse about this finding. At 1500, the nurse takes the patient's temperature, and it is 101.8 degrees. After documenting the findings, the nurse should do which of the following? Select all that apply. A. Administer amoxicillin per the standing order B. Call the physician immediately C. Palpate the patient's right upper quadrant D. Administer acetaminophen E. Place the patient in the Trendelenburg position

B

A post-coronary artery bypass graft patient developed a fever of 38.8° C. The nurse notifies the physician of the elevated temperature because: A. The elevated temperature may lead to profuse sweating. B. It may increase cardiac output. C. It is a sign of cerebral edema. D. It is indicative of hemorrhage.

B, D, E

The nurse is creating a teaching plan for a client diagnosed with pheochromocytoma. Which statement, if made by the client, would require follow-up? Select all that apply. A. "It will be very important to reduce the stress in my life." B. "This condition may cause my glucose to decrease." C. "I will need to monitor my blood pressure closely." D. "If I feel tired, it is okay for me to have an energy drink." E. "Diuretics will be prescribed to help eliminate the fluid I may retain."

B

When caring for an infant during cardiac arrest. Which pulse must be palpated to determine cardiac function? A. Carotid B. Brachial C. Pedal D. Radial

C

When discussing the Denver II test with the parents of a preschooler, which of these statements would indicate that they correctly understood the teaching? A. "This test will tell me whether or not my child's Intelligence Quotient (IQ) is normal." B. "This test will tell me about the motor developmental tasks my child can do today." C. "This test will measure my child's development." D. "This test will let me know if my child's development is normal or not."

D

Which technique is effective for determining and evaluating the effectiveness of the nurse's therapeutic communication and therapeutic communication techniques? A. Performance improvement studies B. ISBAR C. Critical thinking D. Process recording

D

While working in the emergency department, the nurse assesses a 3-day old infant brought in by the mother. The mother states, "My baby is always so sweaty and hot, and just doesn't want to eat! I think something is wrong." The nurse is unable to palpate a femoral pulse but notes +3 brachial pulses. Based on this assessment, which congenital heart defect does the nurse suspect? A. Hypoplastic left heart syndrome (HLHS) B. Patent ductus arteriosus (PDA) C. Transposition of the great arteries (d-TGA) D. Coarctation of the aorta (COA)

D

A nurse is assigned to care for a client who recently underwent a thyroidectomy. The nurse notes that the client has developed peripheral numbness and tingling, muscle twitching, and spasms. Based on this information, the nurse should anticipate administering: A. Thyroid supplements B. Barbiturates C. Antispasmodics D. Intravenous calcium gluconate

A, E

A nurse is caring for a client receiving digoxin. The client's most recent digitalis level was 2.5 ng/mL. The nurse should take which action? Select all that apply. A. Withhold the client's scheduled dose B. Administer the dose, as prescribed C. Assess the client's 24-hour urinary output D. Assess the client's most recent sodium level E. Assess the client's heart rate and rhythm F. Obtain a prescription for an echocardiogram

A Choice A is correct. Guided imagery is a stress-reduction technique that can be done in any place at any time. In fact, this is one of the biggest advantages of this technique. Anytime the patient begins to feel anxious, they can practice guided imagery. Choice B is incorrect. Guided imagery can be done in any position that the patient is most comfortable in. They do not have to by lying down unless they choose to. Choice C is incorrect. It is not necessary for the client's mom or anyone else to be present for guided imagery unless they choose so. Any person, or no one at all, can be present depending on the client's preferences. Choice D is incorrect. Music can but does not have to be played during guided imagery, again it depends on the client's preferences.

A 12-year-old client with chronic asthma exacerbations has decided to try guided imagery as a way to manage the anxiety that is contributing to frequent asthma attacks. Which statement by the client indicates an understanding of this stress-reduction technique? A. "I can do this anytime and anywhere when I feel anxious." B. "I must be lying down to practice guided imagery." C. "My mom will have to be with me any time I try this." D. "I will play music every time I do my guided imagery to make sure it works."

A Choice A is correct. Levonorgestrel (LNG) is available over the counter for emergency contraception. This medication is indicated to be used up to 72 hours following unprotected intercourse, where pregnancy could be possible. It may be used off-label up to 120 hours following the event. This medication works by postponing (or inhibiting) ovulation. Choices B, C, and D are incorrect. These medications are not indicated as emergency contraception. ➢ Tamoxifen is indicated for hormone receptor-positive breast cancer. This medication is used to prevent breast cancer recurrence. ➢ Finasteride is indicated for benign prostatic hyperplasia. This medication is highly teratogenic and should be handled with gloves by pregnant women. ➢ Methotrexate is indicated for ectopic pregnancy that has not ruptured. This may be an alternative to surgery. Several methods may be used for emergency contraception (EC). The most commonly used are - Levonorgestrel (LNG) is an oral pill that may be taken 72-120 hours following unprotected intercourse. This medication is available without a prescription. Abdominal cramping, fatigue, vaginal bleeding/spotting. Copper IUD is the most effective form of EC because it may be placed for up to five days following unprotected intercourse. Some prescribers may place this device ten days after unprotected intercourse if specific criteria have been met. A significant disadvantage is that a practitioner must place this device. The client may keep this device inserted to prevent future pregnancies. The current guideline is that this device must be removed and replaced after ten years.

A client presents to the clinic asking the nurse about emergency contraception. The nurse anticipates that the primary healthcare provider (PHCP) will prescribe which medication? A. Levonorgestrel B. Tamoxifen C. Finasteride D. Methotrexate

D Choice D is correct. Respiratory distress is not caused by untreated depression during pregnancy. Choices A, B, and C are all incorrect. Teratogenicity, non-adherence to prenatal care, and tobacco use are associated issues with untreated depression in pregnancy.

A depressed pregnant patient is being seen in the clinic. Her physician has suggested that she try an anti-depressant to treat the condition. But the patient is nervous. The nurse should explain that all of the following are possible outcomes of untreated depression, except: A. Teratogenicity B. Non-adherence to prenatal care C. Tobacco use D. Respiratory distress post-birth

D

A nurse is caring for a client receiving nitroglycerin. It is essential to monitor the client's A. Temperature B. Respirations C. Urinary output D. Blood pressure

A Choice A is correct. Mineral oil is contraindicated in pregnancy as it decreases nutrient absorption in the mother. Choice B is incorrect. Sleeping in a side-lying position removes the weight of the fetus on the superior and inferior vena cava, promoting venous return and decreasing venous pressure. Choice C is incorrect. Increasing fiber and water intake promote the formation of bulkier stools. Preventing constipation and relieving rectal pain. Choice D is incorrect. Cold compresses relieve pain by vasoconstriction of the hemorrhoids.

An 11-week pregnant client is complaining to the nurse about her hemorrhoids. The nurse understands that hemorrhoids occur because of pressure on the rectal veins from the bulk of the growing fetus. All of the following are measures to alleviate hemorrhoid pain, except: A. Instruct the client to use mineral oil to soften her stools. B. Rest in a side-lying position daily. C. Increase the client's fiber and water intake. D. Apply a cold compress to the area

D Choice D is correct. It is the responsibility of the nurse to evaluate and check if the delegated tasks to the LPN have been performed. Choice A is incorrect. The LPN cannot assess a client. This is a task for the RN. Choice B is incorrect. The child has just undergone a cleft palate repair. There is a risk for the child to damage his incision site and aspirate if he/she is fed by untrained personnel. This task is for the RN. Choice C is incorrect. Demonstrating a procedure to the mother is similar to educating or teaching the client. The LPN is not allowed to perform teaching/education.

An RN is in charge of the unit with an LPN. Which situation indicates proper delegation of tasks by the RN? A. The RN delegates to the LPN to check the circulation of the child with a forearm cast. B. The LPN is tasked to feed a one-year old that just had a cleft palate repair. C. The LPN demonstrates urinary catheterization to the mother of a child with neurogenic bladder. D. The RN checks if the LPN completed all delegated tasks.

A Choice A is correct. At 1-year-old, children should be beginning to walk. Hospitalization during this age could delay this stage of development. Choice B is incorrect. The patient should just be learning to walk at this age, not running. Choice C is incorrect. The child should be sitting up by six months of age. Choice D is incorrect. The child should already be crawling before age 1.

Hospitalization may affect or delay the progression of which physical development of a 1-yr-old patient? A. Walking B. Running C. Sitting D. Crawling

C Choice C is correct. Phosphorous and calcium have an inverse relationship, meaning that as one level rises, the other decreases. Since this patient has hypocalcemia or low calcium, decreasing serum phosphorus through phosphate secreting medications will inversely increase serum calcium. Choices A, B, and D are not correct. These would not occur.

The nurse is administering phosphate excreting medications to her patient with hypocalcemia because she understands what core information about calcium and phosphorous? A. As phosphorous exits the body so does calcium. B. Calcium is managed by the excretion of phosphorous. C. When serum phosphorous decreases, serum calcium increases. D. Phosphorous must be above 4.5 mg/dL before calcium can increase.

A

The nurse is taking care of a patient that was recently rescued from a near-drowning experience. The patient is now having pulmonary edema. The nurse understands that pulmonary edema is the result of which process? A. Water washing out the alveolar surfactant. B. Water introducing bacteria into the lungs and causing infection. C. Decreased intrathoracic pressure in the lungs. D. A sudden change in temperature within the lungs.

A Choice A is correct. The progressive stage occurs when compensatory mechanisms begin to fail. Signs/symptoms of the progressive stage include anasarca (generalized edema), decreased responsiveness, decreased urine output, weak pulses, hypotension, and tachycardia. Choice B is incorrect. The compensatory stage of shock is the first stage, characterized by hypotension, vasoconstriction, decreased blood flow to the lungs, and cold/clammy skin. Choice C is incorrect. The irreversible stage is the final stage of shock, characterized by decreased perfusion due to decreased cardiac output, hypotension, hypoxemia, cyanotic skin, vasoconstriction, bradycardia, and unresponsiveness. Choice D is incorrect. The nonprogressive stage is an alternative term for the compensatory stage.

The patient with septic shock presents with anasarca, weak pulses, decreased urine output, decreased responsiveness, and BP 88/52 mmHg, HR 160. The nurse would recognize these symptoms as indicators of which stage of shock? A. Progressive B. Compensatory C. Irreversible D. Nonprogressive

C Choice C is correct. Family members can become frustrated when clients with Alzheimer's disease lose short-term memory. The nurse should explain to the family member that it's the "short-term memory" that is declining and encourage the client to talk about things that he/she can remember. Choice A is incorrect. During the early stages of Alzheimer's, family members are still trying to learn about and cope with the changes that their loved ones are experiencing. Patience with the family will be more beneficial than the scolding tone that this answer choice portrays. Choice B is incorrect. Early Alzheimer's symptoms are not usually reflective of anxiety. Also, the client is not reliving past experiences because it makes him calm again. Instead, his behavior is expected as Alzheimer's first affects short-term memory. Choice D is incorrect. Reminding an Alzheimer's patient that he is repeating himself will not improve the behavior as his short-term memory is affected. The hippocampus is the structure responsible for creating new memories from experiences. When it is damaged, short-term memory is not possible.

The son of a client with early Alzheimer's disease states, "I'm so tired of hearing Dad talk about the past all the time." What is the nurse's best response? A. "You should be more patient with your father and accepting of his disease." B. "He is quite anxious at this stage. Reliving the past helps him become calm again." C. "He has lost his short-term memory but can still remember events from long ago." D. "Just remind him when he repeats himself and that will reinforce better behavior."

A Choice A is correct. The nurse is a mandatory reporter of any suspected violence and is required by law to report her suspicions. Parents may become upset and confront the nurse when these allegations come to light, but that should not stop the nurse from saying what she has seen. The nurse should remain calm when the parent confronts her and she should state that she is required to report any suspected violence. Choice B is incorrect. This statement is an accusation and could further aggravate the mother. The nurse does not know that she abused her son and should not make statements such as this one. The nurse needs to remain calm and stick to the fact that she is required to report any suspected violence. Choice C is incorrect. This statement is based on emotion, not fact. The nurse should not apologize to the mother. Instead, she should remain calm and inform her that she is required to report any suspected violence. Choice D is incorrect. This is not an appropriate statement. By telling the mother, she doesn't want to see her; she could further aggravate the mother and place herself in a dangerous situation. The nurse needs to remain calm and stick to the facts. If she feels like she is not safe with the mother, she should move to an open area where others are nearby.

The school nurse is assessing a 12-year old boy who came into her office for a nose bleed. She notices several bruises on his back and forearms that are in various stages of healing. When she asks the boy about them, he is very deceptive. The nurse notifies child protective services of her suspicion. The next day, the boy's mother comes to the nurse's office and yells at her for calling child protective services. Which of the following responses is most appropriate? A. "I am required by law to report any suspected violence." B. "You should have thought about this before you abused your son." C. "I'm so sorry. Please don't take this out on me." D. "Don't talk to me about this. I don't want to see you."

D Choice D is correct. When there is an excessive loss of fluid within the body, dehydration can occur. Dehydration may be caused by acute illness or a chronic disease process. Common symptoms include dry mucous membranes, dark urine, decreased urinary output, confusion, low blood pressure, muscle cramps, and constipation. Choice A is incorrect. Urinary output is decreased with dehydration. Choice B is incorrect. Weight gain and edema are not signs of dehydration. However, weight loss and poor skin turgor are signs of dehydration. Choice C is incorrect. Patients experiencing dehydration will exhibit hypotension, not hypertension.

When assessing for dehydration, the nurse should observe for which of the following? A. Headache and increased urinary output B. Weight gain and edema C. Hypertension and decreased urinary output D. Hypotension, headache, and dry mucous membranes

C Choice C is correct. Topical nitroglycerin is used to help prevent/ treat anginal symptoms in coronary artery disease. To apply nitroglycerin correctly, be sure to rotate the application sites with each application to avoid irritation from the medication. The medication comes with a supply of paper applicators with a small ruler on the paper for proper measurement of the drug. Apply the appropriate amount of ointment on the paper and apply the cream to an area of the skin. Choices A, B, and D are incorrect. Do not rub the cream on the skin until it disappears. Tape the paper into place, and do not cover it with gauze. The cream is usually applied to the chest, back, upper arms, or other torso parts.

You are caring for a patient with a new order for nitroglycerin ointment one inch applied to the skin twice a day to prevent angina. To use nitroglycerin correctly, you know to: A. Apply it only to the upper chest B. Rub the ointment into the skin until it disappears C. Rotate the application sites D. Cover the application site with a gauze dressing

B Choice B is correct. Although all of these findings are abnormal, elevated potassium is a life-threatening finding and must be reported immediately. Acute renal failure can cause a significant imbalance in lab values. Although all of the lab results listed are abnormal, the elevated potassium level is a life-threatening finding. Choices A, C, and D are incorrect. Each of these lab values is abnormal. However, they don't pose a life-threatening finding like answer choice B. A: The average BUN level should be 7 to 20 mg/dL. C: Venous blood pH should be 7.31 to 7.41. D: Normal hemoglobin levels differ based on age, sex, and general health. The normal range for hemoglobin is 13.5 to 17.5 grams per deciliter for men and 12.0 to 15.5 grams per deciliter for women.

Which of the following labs for a client with acute renal failure should be reported immediately? A. Blood urea nitrogen 50 mg/dL B. Serum potassium 6mEq/L C. Venous blood pH 7.30 D. Hemoglobin of 10.3 mg/dL

A Choice A is correct. LPN/LVN education and scope of practice include performing dressing changes and obtaining specimens for wound cultures. Choices B, C, and D are incorrect. Teaching, assessment, and planning of care are complex actions that should be carried out by a registered nurse.

Which of the following nursing actions can an LPN/LVN perform on a patient who has a leg ulcer that is infected with vancomycin-resistant S. aureus (VRSA)? A. Obtain wound cultures during dressing changes. B. Plan ways to improve the client's oral protein intake. C. Assess the risk for further skin breakdown. D. Educate the client about home care of the leg ulcer.

C

Which focus is the nurse most likely to teach for a client with a flaccid bladder? A. Habit training: Attempt voiding at specific time periods. B. Bladder training: Delay voiding according to a pre-scheduled timetable. C. Credé's maneuver: Apply gentle manual pressure to the lower abdomen. D. Kegel exercises: Contract the pelvic muscles.

A, C

Which of the following are complications of acute tubular necrosis (ATN)? Select all that apply. A. Metabolic acidosis B. High thyroxine levels C. Hyponatremia D. Decreased parathyroid levels

A, C

Which of the following are potential causes of metabolic alkalosis? Select all that apply. A. Vomiting B. Diarrhea C. Antacids D. Starvation

A Choice A is correct. This ABG shows metabolic acidosis. The pH is less than 7.35, which is acidosis. The PCO2 is between 35 and 45, which is normal. Lastly, the HCO3 is less than 22, which is acidotic. The HCO3 shows acidosis like the pH, so we know this is metabolic acidosis. Choices B, C, and D are incorrect. These are not the correct acid-base disorder for the patient.

Which of the following is the correct interpretation for the following arterial blood gas? pH: 7.31 PCO2: 40 HCO3: 18 A. Metabolic acidosis B. Respiratory acidosis C. Metabolic alkalosis D. Respiratory alkalosis

C Choice C is correct. The parenting style described is authoritarian. This parent is often described as the 'rigid disciplinarian'. They are highly controlling; they expect always to be obeyed and are still inflexible with the rules. Though these parents may have their child's best interests at heart, they do not support their growing autonomy. Instead, they expect to be obeyed without reason. Choice A is incorrect. Authentic not the parenting style described. An authentic parent sets reasonable limits on behavior, encourages the growing autonomy of their children, and promotes open communication about the rules. These parents would not be inflexible with the regulations; rather, they would explain why the rules work the way they do with their children to promote autonomy and open communication. Choice B is incorrect. Permissive is not the parenting style described. A permissive parent sets few or no restraints, gives their child complete unconditional love no matter the situation, as much freedom as they desire, places no limits, and offers very little guidance. While it is clear to everyone that these parents love their children very much, they are not constructive parents and do not support the growth of their children. Choice D is incorrect. Indifferent is not the parenting style described. An indifferent parent sets no limits for their child, lacks affection for their child, and is rather focused on their own life instead of that of the child. They are utterly indifferent to what their child does. This is very clearly ineffective for raising a healthy child.

Which of the following parenting styles is described as highly controlling, expecting to always be obeyed, and inflexible with the rules? A. Authentic B. Permissive C. Authoritarian D. Indifferent

A, D

Which of the following signs does the nurse know to expect for her 1-year-old patient in heart failure? Select all that apply. A. Diaphoresis B. Weight loss C. Insomnia D. Poor feeding

B, C

Which of the following statements about calcium are true? Select all that apply. A. Calcium increases vitamin D levels. B. 50-70% of serum calcium is ionized in the serum. C. Albumin and calcium levels can be directly correlated. D. Calcium that is bonded to protein can pass through capillary walls.

A, B Choices A and B are correct. The vasodilation properties of a beta-blocker mean that they decrease blood pressure. This is because the beta-blockers are blocking the receptor sites for your catecholamine, so they cannot do their job and cause vasoconstriction (Choice A). Beta-blockers decrease the workload of the heart. This is because of the vasodilation, subsequent decrease in blood pressure, and then fall in afterload. Remember, afterload is the pressure against which the left ventricle must pump. With decreased blood pressure, we reduce afterload. With reduced afterload, the left ventricle does not have to work as hard to pump blood to the body. Therefore, beta-blockers decrease the workload of the heart (Choice B). Beta-blockers block the beta cells of the body. Beta cells are receptor sites for catecholamines, such as epinephrine and norepinephrine. When we block the receptor sites for the catecholamines, they cannot do their job. Catecholamines function to increase everything - increase blood pressure, increase pulse, increase contractility, and cause vasoconstriction. This is because they are fight-or-flight hormones. They get the body excited and ready to go. So, when beta-blockers block them, everything decreases. Your body vasodilates, the heart slows down, and the blood pressure decreases. Choice C is incorrect. Beta-blockers decrease contractility, not increase; this is because they are blocking those beta cell receptor sites for catecholamines such as epinephrine and norepinephrine. The catecholamines work to increase contractility, but they are blocked by the beta-blockers. So, beta-blockers decrease contractility. Choice D is incorrect. Beta-blockers decrease cardiac output, not increase; this is because of the decreased contractility. While the catecholamine receptor sites are blocked, they are unable to cause increased contractility and the contractility of the heart decreases. With decreased contractility comes a decreased stroke volume. Since CO = HR x SV, a reduced stroke volume means a reduced cardiac output.

Which of the following statements are true regarding beta blockers' mechanism of action? Select all that apply. A. Decrease blood pressure B. Decrease workload of the heart C. Increase contractility D. Increase cardiac output

A, B

Which of the following statements are true regarding neural tube defects? Select all that apply. A. Types of neural tube defects include spina bifida occulta, spina bifida cystica, meningocele, and myelomeningocele. B. The nurse should protect the exposed sac by covering with a sterile, moist, non-adherent dressing. C. Left-lateral is the optimal position to minimize tension on the sac. D. Neurological deficits are always present in patients with neural tube defects.

A, B, C

Which of the following statements regarding mass casualty events are true? Select all that apply. A. Mass casualties are events that overwhelm local medical capabilities. B. When a mass casualty occurs, there is a need to increase the staff at the hospital. C. Many local agencies will collaborate to handle a mass casualty situation. D. An example of a mass casualty event is a fight between visitors in the intensive care unit.

A, B Choices A and B are correct. Osmosis is an essential principle upon which peritoneal dialysis functions. Osmosis is the passive movement of solvents, such as water, across a permeable membrane. The peritoneum is a permeable membrane. (Choice A). Diffusion is an essential principle upon which peritoneal dialysis functions. Distribution is the passive movement of solutes across a membrane. Solutes diffuse from an area of higher concentration to an area of lower concentration, across the peritoneum, until there is an equal amount of each on both sides of the membrane (Choice B). Choice C is incorrect. The oncotic pressure is a form of osmotic stress induced by proteins in a blood vessel's plasma that displaces water molecules. This is not an essential principle upon which peritoneal dialysis is based. Choice D is incorrect. Osmotic pressure is the pressure that would have to be applied to a pure solvent to prevent it from passing into a given solution by osmosis. This is not an essential principle upon which peritoneal dialysis is based.

While orienting a new graduate nurse in the ICU, you take care of a patient scheduled for peritoneal dialysis. Which of the following principles do you explain to the new graduate about peritoneal dialysis functions? Select all that apply. A. Osmosis B. Diffusion C. Oncotic pressure D. Osmotic pressure

A

Intravenous therapies often consist of electrolyte replacement therapies. Select the electrolyte that is accurately paired with one of its functions. A. Sodium: The control and management of circulating blood volume. B. Bicarbonate: The regulation of extracellular fluid. C. Chloride: The regulation of plasma protein. D. Calcium: The metabolism of fats, carbohydrates, and proteins.

C

A client is scheduled to undergo an outpatient diagnostic bronchoscopy. The client's spouse transported the client to the outpatient clinic and will provide all care following the client's discharge. Out of the following nursing diagnoses, which is the most appropriate for this client? A. Risk for impaired skin integrity related to immobilization B. Risk for infection related to an invasive procedure C. Risk for bleeding related to diagnostic bronchoscopy D. Lack of knowledge regarding postoperative care related to inexperience with diagnostic bronchoscopy as evidenced by frequent queries about the postoperative routine

C

The ER nurse assesses a patient for tactile fremitus. Which would be the correct way to assess for this abnormal finding? A. Percuss the apices in the supraclavicular areas. B. Instruct the patient to breathe deeply while auscultating both sides of the lungs. C. Ask the patient to say "ninety-nine" while palpating the chest. D. Place the hands along the anterolateral wall with thumbs pointing toward the xiphoid process.

C

The maternal health nurse is taking the history and physical for a pregnant woman in her second trimester. She has several mysterious bruises on her arms and appears isolative. The nurse would provide vigilant surveillance for which common occurrence in pregnancy? A. Chronic depression B. Physiological anemia C. Domestic violence D. Acute insomnia

D

The medical-surgical nurse is assigned to the intensive care unit (ICU). The nurse accepts the hand-off reports and indicates they are not qualified to provide the necessary care. It would be appropriate for the medical-surgical nurse to take which action? A. Refuse to provide any client care B. Document the concern in the client's medical record C. Notify the primary healthcare provider (PHCP) D. Report the concerns to the nursing supervisor

B

The nurse cares for a client immediately following a shoulder reduction procedure with moderate sedation. The nurse assesses the client as restless and irritable. The nurse should take which priority action? A. Assess the client for pain B. Assess the client's oxygen saturation C. Assess the client with the Glasgow Coma Scale (GCS) D. Assess the client's lung sounds

A, B, D, E

The nurse is caring for a client who presents with hyperglycemia. Which of the following findings are expected? Select all that apply. A. Blurred vision B. Increased urinary output C. Cool and clammy skin D. Tachycardia E. Orthostatic hypotension

45%

The nurse is caring for a client who sustained a full-thickness burn to his anterior torso, back, and bilateral anterior arms. Using the rule of nine's, calculate the total body surface area (TBSA) burned. Fill in the blank

A, B, E

The nurse is caring for a client whose most recent serum sodium level was 152 mEq/L. Which of the following signs and symptoms can be attributed to the client's sodium level? Select all that apply. A. Lethargy B. Dry mucous membranes C. Tachypnea D. Cyanosis E. Excessive thirst

B

The nurse works on a medical/surgical unit and cares for a patient receiving digoxin and furosemide. Which of the following, if reported by the patient, must be assessed immediately? A. Night sweats and headache. B. Vomiting and halos around lights. C. Stomach upset and headache. D. Low blood pressure and dark urine.

A Choice A is correct. Intussusception generally occurs as a result of a blockage in the intestines, which results in the telescoping of one portion of the bowel into another part of the colon. This disorder occurs more frequently in children, often males. Choice B is incorrect. Intussusception is the telescoping of the intestine and does not have to do with alveoli. Choice C is incorrect. Twisted bowel is simply a twist or loop in the gut and is not known as intussusception. Choice D is incorrect. Rectal prolapse occurs from chronic constipation and weakened anal sphincter.

The pediatric nurse is reading the chart for a newly admitted child suffering from intussusception. The nurse knows that this disorder is characterized by: A. The telescoping of one area of the intestines into another B. The absence of alveoli in one segment of the lobe C. A twisted colon D. The prolapse of the rectum

B

A 68-year-old woman arrives at the emergency department after feeling dizzy. After assessing the patient, the nurse notices hypotension, muffled heart tones, and jugular venous distention. What does the nurse suspect that this patient has? What is this triad called? A. Pericarditis; Cushing's triad B. Pericardial tamponade; Beck's triad C. Increased ICP; Cushing's triad D. Pleural effusion; Beck's triad

C

During a physical assessment, the nurse inspects the patient's abdomen. What assessment technique would the nurse perform next? A. Percussion B. Palpation C. Auscultation D. Whichever is most comfortable for the patient

A

Following a pregnant client's report of persistent nighttime leg cramps over the past week, it would be appropriate for the nurse to instruct the client to increase their dietary intake of: A. Whole grains, nuts, egg yolks B. Almonds, sweet potato, avocado C. Lentils, peas, nuts D. Carrots, tomatoes, squash

C

Select the category of pain medication that is accurately paired with the level of pain medication indicated for it AND an example of a pain medication that is included in this category of pain medication. A. Co-analgesic medications: Severe pain: A tricyclic antidepressant medication B. Opioid analgesic medications: Severe pain: Tramadol C. Opioid analgesic medications: Moderate pain: Tramadol D. Non-opioid analgesic medications: Moderate pain: Ibuprofen

D Choice D is correct. Making assumptions or generalizations about a patient's spiritual needs based on ethnic or religious affiliation is almost sure to be an oversimplification. The nurse should be able to identify similarities and differences among the cultural beliefs of the patients. Just because a patient belongs to a certain culture or ethnicity, it is incorrect to generalize their spiritual needs. Choices A, B, and C are incorrect. Ordering a specific diet as per the patient's specific cultural or religious preference is certainly warranted. However, generalizations cannot be made here either, and knowing the patient's specific preference will help the nurse cater to the patient's dietary or spiritual needs. Communicating with the family and performing a spiritual consult should also be done at the patient's request. While identification of cultural similarities and differences among the patients can help guide these processes, these are not the primary objectives. The primary objective is to avoid making assumptions.

The primary objective in identifying similarities and differences among cultural beliefs of a patient is to: A. Communicate with the family B. Make sure the proper diet is ordered C. Perform a spiritual consult D. Avoid making assumptions

A

What is the most important factor to consider when assessing a home health patient on the risk of falls? A. Correct illumination of the environment B. Amount of regular exercise C. The resting pulse rate D. Status of salt intake

B

When instructing a post-surgical patient with an abdominal incision on deep breathing and coughing, the nurse explains that the patient should be sitting up for these activities because: A. It is physically more comfortable for the patient B. Helps the patient to support their incision with a pillow C. Loosens respiratory secretions D. Allows the patient to observe their area and relax

A

Which action taken by the school nurse will have the most impact on the incidence of infectious disease in the school? A. Ensure that students are immunized according to national guidelines. B. Provide written information about infection control to all patients. C. Make soap and water readily available in the classrooms. D. Teach students how to cover their mouths when coughing.

A, C Choices A and C are correct. Stage II pressure ulcers occur when the epidermis is lost as well as part of the dermis (Choice A). Stage III pressure ulcers expose subcutaneous fat but do not extend deep enough to expose the bone and muscle. That would be considered a Stage IV pressure injury (Choice C). Choice B is incorrect. Stage I pressure ulcers do not involve any loss of tissue. The epidermis remains intact, but it is reddened and does not blanch. Choice D is incorrect. Stage IV pressure ulcers expose bone and muscle. If the base of the wound is covered by slough or eschar, the nurse can't assess how deep the pressure injury goes. Therefore the pressure ulcer is considered unstageable.

Which of the following statements regarding pressure ulcers are true? Select all that apply. A. In a stage II pressure ulcer, part of the dermis and epidermis are lost. B. In a stage I pressure ulcer, there is a loss of integrity of the epidermis only. C. In a stage III pressure ulcer, there is a deep tissue injury that can expose fat. D. In a stage IV pressure ulcer, the base of the wound is covered by eschar.

C Choice C is correct. It is widely accepted that a finding of a single transverse palmar crease on the palm - often referred to as a simian line or simian crease - is often observed in a wide range of chromosomal defects, including, but not limited to, Down syndrome, congenital limb deficiency, trisomy 13/18/21, 4p, 18q, etc. Although this finding does not in and of itself render a diagnosis of a chromosomal disorder, this finding by the nurse would necessitate the need to alert the newborn's primary health care provider (HCP), as genetic and chromosomal testing will likely need to be performed. Choice A is incorrect. Fontanels are one of a newborn's skull's most prominent anatomical features, helping to facilitate the movement and molding of the newborn's cranium through the birth canal during labor. The diamond-shaped soft area present at the top of the newborn's head is the anterior fontanel (also commonly referred to as a fontanelle), the largest of the six fontanels present at birth. The anterior fontanel typically fuses between 12 and 18 months of age. Choice B is incorrect. This finding is indicative of vernix caseosa. At birth, a newborn's skin may be covered with vernix caseosa, a grayish-white, cheese-like substance composed of a mixture of sebum and desquamating cells. If it is not entirely removed during the post-birth bath, the newborn's skin will absorb any remaining vernix caseosa within 24 to 48 hours. Choice D is incorrect. Acrocyanosis is often seen in healthy newborns and refers to the peripheral cyanosis around the mouth and the extremities (hands and feet). Unlike other causes of peripheral cyanosis with significant pathology (e.g., septic shock), acrocyanosis is a harmless condition caused by benign vasomotor changes that result in peripheral vasoconstriction and increased tissue oxygen extraction. Additionally, acrocyanosis is further differentiated from peripheral cyanosis as acrocyanosis occurs immediately after birth in healthy infants and is a common finding, at times persisting for 24 to 48 hours. LEARNING OBJECTIVE While working as a nurse in a nursery, recognize that a single crease on the palm of a newborn would be the nursing assessment finding that would necessitate further investigation. ADDITIONAL INFO Using simian creases to screen for certain known congenital chromosomal disorders and syndromes could be useful in conjunction with other diagnostic practices. Simian creases can be examined quickly without causing physical pain or incurring expensive medical bills. However, the detection of simian creases should not be used independently for diagnosing known associated chromosomal disorders and/or other disorders, but rather in combination with other diagnostic tests. A consensus statement based on the results of a study conducted by the National Association of Neonatal Nursing (NANN) and the Association of Women's Health Obstetrical and Neonatal Nursing (AWHONN) directed "removal of all vernix is not necessary for hygienic reasons" and "vernix may provide antibacterial promotion and wound healing."

While working in the nursery, a nurse assesses a newborn born less than two hours ago. Which of the following findings by the nurse would necessitate further investigation? A. A diamond-shaped soft area present at the top of the newborn's head B. Greasy, white substance that resembles cheese on the newborn's neck, back, and thighs C. A single crease on the palm D. Acrocyanosis

B

You receive the change-of-shift report for an infant whose family has just been informed of the infant's cystic fibrosis diagnosis. As the nurse caring for this pediatric client and the family, which of the following should you prioritize? A. Arrange and schedule a follow-up appointment with a pediatric pulmonologist B. Provide emotional support for the family C. Arrange for financial assistance D. Arrange for parental genetic testing, as the parents mention they want another child soon

A Choice A is correct. Hyperventilating can cause respiratory alkalosis. This is because there the body is blowing off too much CO2. CO2 is an acid, so when the body is loosing too much of it, the client can become alkalotic. Choice B is incorrect. These values represent typical ABG values, which would not be expected in a patient who is hyperventilation. Choice C is incorrect. These values represent respiratory acidosis, which is not caused by hyperventilation. Respiratory acidosis is more likely to occur when the patient is hypoventilating, and retaining too much CO2. Common causes of this are an overdose or respiratory depression. Choice D is incorrect. These values represent metabolic alkalosis, which would not be expected in the patient who is hyperventilating. Because it is a change in CO2 causing the pH to shift, the cause of the imbalance is respiratory, not metabolic.

A 16-year old patient injures her ankle on the soccer field. She is taken to the emergency department by ambulance. In the ambulance, she starts hyperventilating. Upon arrival to the waiting room, an arterial blood gas is drawn. What values will most likely appear on the results? A. pH: 7.55, CO2: 22, HCO3: 24 B. pH: 7.35, CO2: 39, HCO3: 26 C. pH: 7.32, CO2: 47, HCO3: 25 D. pH: 7.55, CO2: 42, HCO3: 34

D Choice D is correct. The nurse's primary responsibility is patient safety. For this deaf and blind patient, it is critical to provide secure environment. According to Maslow's hierarchy of needs, physiological needs and thereafter, safety needs should be prioritized in that order. Visual impairment has been associated with falls that often result in fractures, and dislocations. A patient with visual impairment may experience disorientation as a consequence of being in a strange hospital environment. Certain important interventions the nurse can undertake in providing a secure environment for a deaf-blind client include: escorting the patient around the new environment as and when required. This will help meet the need for safety, promote some orientation and instill a feeling of security in the patient. orienting patient to layout of room, restrooms, location and operation of call button, telephone, television, and environmental controls. Such orientation helps prevent accidents. The nurse must also provide adequate supervision when patient needs to visit the toilet. Other measures include placing the bedside locker on the side most appropriate for the individual patient, and placing the the call within easy reach. orienting patient to treatment room and supplies. orienting patient to lounges, recreation rooms, and nursing station in relationship to patient's room. communicating evacuation/rescue plans; orientation to fire alarm pull boxes, fire extinguisher, and emergency exits training in self-care and use of medical equipment. assisting with feeding, toileting, bathing, or dressing (only if required). Choices A, B, and C are incorrect. The nurse should make other care team members aware of patient's visual and hearing deficits using a sign placed above the bed, in report, and on medical record. Communication with the supervisor regarding safety concerns, and updating the patient on social environment are all necessary, however, patient safety needs (providing a secure environment) takes the utmost priority.

A 40-year-old patient who is blind and deaf has been admitted to the medical floor. What is the nurse's primary responsibility for this patient? A. Make others aware of the patient's deficits B. Communicate with the nursing supervisor any patient safety concerns C. Continuously update the patient on the social environment D. Provide a secure environment for the patient

C Choice C is correct. Yogurt is a dairy product and therefore contains lactose. Breastfeeding mothers with infants who are lactose intolerant should avoid dairy products such as cheese, milk, and yogurt. Choice A is incorrect. Leafy greens do not contain lactose and do not need to be avoided by the mother nursing a lactose intolerant infant. Choice B is incorrect. Red meat does not contain lactose and does not need to be avoided by the mother nursing a lactose intolerant infant. Choice D is incorrect. Wheat rolls generally do not contain lactose and do not need to be avoided by the mother nursing a lactose intolerant infant.

A breastfeeding mother is struggling to care for her infant with lactose intolerance. Which of the following foods should the mother avoid? A. Leafy greens B. Red meats C. Yogurt D. Wheat rolls

A Choice A is correct. This client displays signs and symptoms of an ST-segment elevation myocardial infarction (STEMI). In a STEMI, myocardial necrosis is occurring, with the client exhibiting ECG changes showing ST-segment elevation not quickly reversible by nitroglycerin administration. The primary focus should be on improving myocardial oxygenation and reducing cardiac workload, as these measures will reduce further expansion of the myocardial necrosis. Choice B is incorrect. Based on Maslow's hierarchy of needs, the needs to confirm the suspected STEMI diagnosis and prevent complications are not prioritized over the need to improve myocardial oxygenation and reduce the cardiac workload. Choice C is incorrect. Based on Maslow's hierarchy of needs, pain relief and anxiety reduction are not prioritized over the need to improve myocardial oxygenation and reduce the cardiac workload. Choice D is incorrect. Providing a quiet, non-demanding environment to reduce anxiety is not a nursing intervention that should be prioritized above the need to reduce cardiac workload and improve myocardial oxygenation based on Maslow's hierarchy of needs. LEARNING OBJECTIVE Recognize that when a client presents to the emergency department with symptoms indicative of a myocardial infarction, the nurse should prioritize improving myocardial oxygenation and reducing cardiac workload according to Maslow's hierarchy of needs. ADDITIONAL INFO An acute myocardial infarction (MI) is myocardial necrosis resulting from acute obstruction of a coronary artery. Diagnosis occurs via ECG and cardiac markers. Immediate treatment typically includes oxygen, antianginals, antiplatelet drugs, and anticoagulants. For those with an ST-segment elevation MI (STEMI), these clients should undergo an immediate angiography with percutaneous coronary intervention (PCI); if an immediate PCI is not unavailable, administer fibrinolytics. For clients with non-ST-segment elevation MI who are stable, perform an angiography within 24 to 48 hours; for those who are unstable, an immediate angiography with PCI should be performed. Following recovery, clients should be initiated or continued on antiplatelet medications, beta-blockers, angiotensin-converting enzyme inhibitors, and/or statins.

A client arrives at the emergency department (ED) complaining of substernal chest pain. An ECG shows ST-segment elevation, and the client's cardiac troponin level is found to be elevated. Which of the following should be the nurse's primary focus? A. Reducing cardiac workload and improving myocardial oxygenation B. Confirming the suspected diagnosis and preventing complications C. Pain relief and reduction of anxiety D. Providing a quiet, non-demanding environment and reducing anxiety

C

A client suddenly develops syndrome of inappropriate antidiuretic hormone (SIADH) after undergoing cranial surgery. Which of the following would be an expected finding? A. Peripheral edema B. Excessive urine production C. Normal or slightly increased blood pressure D. A low urine specific gravity

D

A common prerenal cause of acute kidney injury is: A. Nephrotoxicity B. Bladder cancer C. Contrast media D. Hypovolemia

B

A male client with chronic renal failure has questions regarding the effects of his kidney disease on his sexual activity. Which of the following is a sexual complication of chronic renal failure? A. Retrograde ejaculation B. Decreased testosterone C. Hypertrophy of the testicles D. Feelings of euphoria

D

A middle-aged man comes into the clinic for a check-up. His pulse is 49 beats per minute and the client reports that he has gained 20 pounds in the past two months. The nurse also notices that his skin is cool to touch and that he has three layers of clothing. The nurse suspects hypothyroidism. Which nursing diagnosis should be of the highest priority? A. Fatigue B. Activity intolerance C. Hypothermia D. Decreased cardiac output

A, C, D

A newly admitted client is in septic shock. This client has a high risk for injury related to a known clotting disorder. Which of the following are appropriate interventions that should be added to the nursing care plan for this client? Select all that apply. A. Administer packed RBCs, if ordered. B. Place a piece of gauze over a venipuncture site and dress with paper tape C. Obtain an order for a stool softener. D. Encourage the client to rinse his mouth with mouthwash and use an oral sponge to brush the teeth.

D Choice D is correct. Sexual pleasure is heightened during the second trimester of pregnancy. In the second trimester, most women experience significant relief from the discomforts of early pregnancy (nausea and vomiting, breast tenderness). The uterus is not too large to interfere with comfort and rest. The second trimester is also the time when pelvic organs are congested with blood, increasing pleasure in sexual activities. Choices A and B are incorrect. As long as risk factors such as preterm labor or incompetent cervix are not present, intercourse should not harm the pregnancy. Sexual intercourse should not be a cause of concern even in the third trimester unless risk factors such as preterm labor or placenta previa are present. Choice C is incorrect. Many women experience changes in sexual desire at different stages in pregnancy, depending on their general sense of well-being and the presence of certain discomforts brought about by the pregnancy. It is not the same throughout pregnancy.

A nurse at an obstetric clinic has conducted a teaching class on sexuality during pregnancy. Which of the following comments from a participant would indicate that the teaching has been effective? A. "At around the time I would normally have my period, I should abstain from intercourse." B. "I should no longer have sex during the last trimester of pregnancy." C. "My sexual desire will remain the same for the entire pregnancy." D. "The best time to enjoy sex is in the second trimester."

B Choice B is correct. The injection should be given within 72 hours after birth. RhoGAM is administered intramuscularly within 72 hours after birth to prevent sensitization to the Rh factor in an Rh-negative woman with an infant who is Rh-positive. This injection will prevent hemolytic disease in subsequent pregnancies. Each vial of RhoGAM is cross-matched to a specific woman. The nurse must do all appropriate checks for patient identification to avoid an error in administration. Choices A, C, and D are incorrect. Any history of a systemic allergic reaction to human immunoglobulins is a contraindication for the RhoGAM injection. RhoGAM is administered to an Rh-negative female. The injection should be withheld in a patient who has an elevated temperature.

A nurse has received orders to administer a RhoGAM injection IM to a postpartum patient. Which situation is NOT a contraindication for administration of this injection? A. Administration to a patient who has a history of a systemic allergic reaction to preparations containing human immunoglobulins. B. Administration of the injection within 72 hours after delivery. C. Administration to an Rh-positive female patient. D. Administration to a patient with an elevated temperature.

D Choice D is correct. A central line is a significant risk factor for a client to develop a central line-associated bloodstream infection (CLABSI). This occurs because of suboptimal sterile technique during insertion and/or inappropriate dressing changes. Additionally, TPN is a risk factor as the high glucose content makes the client more likely to develop a bacterial or fungal infection. TPN increases the risk for a CLABSI compared to solutions such as 0.9% saline. Choices A, B, and C are incorrect. A client withdrawing from alcohol has a risk for delirium tremens but not a risk for an infection. Further, being malnourished increases the risk of impaired skin integrity but not an infection. Methylprednisolone is a corticosteroid that, if used intermediate to long term, may increase the risk for infection. Asthma itself is a chronic disease but not one that raises the risk for infection. An external urinary catheter is preferred over an internal catheter as this substantially decreases the risk for cystitis. This is a non-invasive way to collect urine. Urinary incontinence may raise the risk factor for cystitis, but this depends on the type (stress, urge, overflow). Standard precautions are executed for all clients and consist of appropriate hand hygiene. By adhering to standard precautions, the nurse may reduce the risk of a client getting a healthcare-acquired infection. Other strategies that a nurse may employ to reduce the risk of infection include - - Chlorhexidine baths for those immobile or having a central line. - Minimizing the use of internal urinary catheters. - Surgical asepsis when completing a central line dressing change. - Absolute adherence to appropriate hand hygiene.

A nurse is conducting infection control assessments on the nursing unit. Which client is at the greatest risk for infection? A client A. withdrawing from alcohol and is malnourished. B. receiving methylprednisolone for an asthma exacerbation. C. has an external urinary catheter device for urinary incontinence. D. receiving total parenteral nutrition (TPN) via a central line

43 mL

A nurse is preparing to administer vancomycin to a child. The order is for 50 mg/kg/day in three divided doses. The client weighs 13 kg. The medication label indicates vancomycin 500 mg in 100 mL of 0.9% saline. How many mL will the nurse administer per dose

A, C, D, F

A nurse is reviewing prescriptions for assigned clients. Which prescriptions require follow-up with the primary healthcare provider? A client with Select all that apply. A. congestive heart failure prescribed diltiazem. B. hypertension prescribed clonidine. C. diabetes insipidus prescribed hydrocortisone. D. pulmonary emboli prescribed clopidogrel. E. atrial fibrillation prescribed amiodarone. F. bacterial cystitis prescribed valacyclovir.

B Choice B is correct. Nurses need to be aware of the patient's needs even if they do not pertain to the reason for hospitalization and treatment. Observation is a crucial nursing skill. The nurse should always be alert for any changes in a patient's condition, regardless of the initial diagnosis. Being aware of the patient's status will equip the nurse to be a better advocate for patients and to request referrals when concerns or issues arise during care. Choice A is incorrect. Any changes in a patient's status should be reported, even if it has nothing to do with the reason for admission. Choice C is incorrect. A referral to go to the hospital gym is not necessary. Physical Therapy can assist the client with balance and gait issues. Choice D is incorrect. The nursing care plan should include safety measures related to gait/balance impairment. However, gait training will be provided by physical therapy.

A patient being treated for hypertension is assessed by the nurse and found to have poor gait and impaired balance. What would the nurse's appropriate action be? A. Do nothing as this has nothing to do with why the patient was hospitalized. B. Speak with the attending physician about his concerns and request a referral to physical therapy. C. Speak with the attending physician about his concerns and request a referral for the patient to go to the hospital gym. D. Add this issue to the nursing care plan and have daily gait/balance training as an intervention.

C Choice C is correct. Bowel prep is necessary to make sure the x-rays are bright and bowel contents do not obstruct viewing of urinary structures. An IV pyelogram is an x-ray that is used to view the urinary structures. Choice A is incorrect. A full bladder is unnecessary for the test to be successful. Choice B is incorrect. Although the technician should be alerted if any uncomfortable sensations occur, allergies should be checked before the test is administered. Choice D is incorrect. It is not necessary to lie down after the test is performed.

A patient is scheduled for an IV pyelogram. He asks the nurse what he needs to do to prepare for the test. The correct response is: A. "You need to have a full bladder for the test to be successful." B. "You need to alert the technician if you feel any burning after the dye is injected." C. "You will receive a bowel preparation before the test can be performed." D. "You must lie on your back for four hours after the test is performed."

C Choice C is correct. Occupational therapists are excellent resources for helping patients suffering from gait and movement problems. Occupational therapists help patients transition from their hospital life to their homes. While physical therapists are mostly involved in specific gait related issues, occupational therapists also assist to help improve functional mobility so that the patients can perform their activities of daily life ( ADL). Choice A is incorrect. Case managers work with patients and their families to organize their care and to discuss resources available outside of the hospital to meet the patient's needs. They do not resolve gait issues. Choice B is incorrect. While they work with a patient to treat various disorders, nurse practitioners are best utilized to prescribe treatments and monitor a full caseload. They are not the best option to help with physical movement. Choice D is incorrect. A respiratory therapist focuses on improving a patient's oxygen saturation. There may be circumstances where an occupational therapist and respiratory therapist may work together; for example, a patient with COPD who has a low oxygen saturation and is deconditioned from a long-term hospital stay.

A patient recovering from a transient ischemic attack can walk but is having difficulty going upstairs. What professional should visit them to help work through this issue? A. Case manager B. Nurse practitioner C. Occupational therapist D. Respiratory therapist

A Choice A is correct. The charge nurse should consult the infection control nurse for patient placement alternatives, as the pediatric client with pulmonary tuberculosis requires airborne isolation. Choice B is incorrect. The disease is transmittable through airborne droplets. If these two pediatric clients are placed in the same room, the uninfected child could acquire the TB infection through airborne droplets. Choice C is incorrect. Despite being in an airborne isolation room with one another, the TB pediatric client and the pediatric client positive for varicella are a risk to one another. These two clients should not be placed in the same room, as both illnesses are transmitted similarly, putting both clients at risk. Choice D is incorrect. Before refusing the admission, the charge nurse should consult the infection control nurse to discuss available options. If the issue remains unresolved, the charge nurse should also escalate the issue in an attempt to rectify the matter before refusing the pediatric client's admission. Tuberculosis is a chronic, progressive mycobacterial infection, often with an asymptomatic latent period following the initial infection. Tuberculosis most commonly affects the lungs. Symptoms include productive cough, fever, weight loss, and malaise. Treatment is with multiple antimicrobial drugs (i.e., isoniazid, rifampin, pyrazinamide, and ethambutol) given concurrently. In airborne infection isolation (AII), the client is placed in an AII room. The main characteristics of AII rooms are that they have negative air pressure relative to the hall and 12 or more air exchanges per hour, of which at least two are outside air (for some older structures, six or more per hour are acceptable).

A pediatric client with pulmonary tuberculosis (TB) is scheduled to be admitted to the pediatric unit when the charge nurse learns the remaining private room on the unit was filled on the prior shift. No other pediatric TB clients are currently admitted. What is the most appropriate action for the charge nurse? A. Contact the infection control nurse B. Room the client with an uninfected client 6 feet apart C. Place the client with the varicella client currently in the airborne isolation room 6 feet apart D. Refuse to admit the pediatric TB client

A

A post-adrenalectomy client is admitted to the ICU and is on IV hydrocortisone. Which nursing intervention should be included in the client's plan of care? A. Monitor blood glucose levels frequently. B. Keep the client supine for 24 hours. C. Discontinue hydrocortisone once vital signs become stable. D. Educate the client on how to properly clean his wound at home.

D

A pregnant woman with preexisting hypertension is being seen in the clinic. Her blood pressure continues to rise despite attempting first-line therapy with anti-hypertensives. Which of the following medications will be used for the prenatal patient resistant to other blood pressure-lowering medications? A. A calcium channel blocker B. Methyldopa C. Labetalol D. Hydralazine

A Choice A is correct. Abruptio placentae should be considered in pregnant clients who have experienced abdominal trauma. A sign that concealed hemorrhage has occurred is the rapid increase in uterine size along with rigidity. Choice B is incorrect. Ectopic pregnancy is the implantation of a pregnancy in a site other than the endometrial lining of the uterine cavity (i.e., fallopian tube, uterine cornua, cervix, ovary, or abdominal or pelvic cavity). Ectopic pregnancies cannot be carried to term and eventually rupture or involute. Early symptoms and signs include pelvic pain and vaginal bleeding. Hemorrhagic shock can occur with rupture. The anatomic structure containing the fetus usually ruptures between 6 to 16 weeks of gestation. There is no correlation between abdominal trauma and ectopic pregnancies. Choice C is incorrect. Placenta previa is the implantation of the placenta over or near the internal os of the cervix. Placenta previa typically manifests as painless vaginal bleeding after 20 weeks' gestation, with the source of bleeding in placenta previa being maternal. Abdominal trauma is not associated with placenta previa. Choice D is incorrect. Uterine rupture is a spontaneous tearing open of the uterus that may result in the fetus floating in the abdomen. Uterine rupture causes the client severe, constant pain in the abdomen and an abnormally slow heart rate in the fetus. Here, the fetal heart rate is not abnormally slow. LEARNING OBJECTIVE In a client in the 30th week of gestation undergoing evaluation after falling down a flight of stairs with a fetal heart rate (FHR) of 167 beats per minute and all other vital signs stable, recognize abruptio placentae as the potential obstetric emergency must be excluded. ADDITIONAL INFO Normal ultrasonographic findings do not rule out abruptio placentae. The severity of symptoms and signs of abruptio placentae depends on the degree of separation and blood loss. Abruptio placentae typically manifests as uterine bleeding and uterine pain or tenderness; bleeding varies in volume and acuity and, if the abruption is concealed, may be absent. Manage with prompt cesarean delivery if maternal or fetal stability is threatened or pregnancy is at term. Consider vaginal delivery if the mother and fetus are stable and the pregnancy is at term.

A woman in her 30th week of gestation was brought in for evaluation after falling down a flight of stairs. Upon evaluation, the health care provider (HCP) notes a rigid, board-like abdomen. The fetal heart rate (FHR) is currently 167 beats per minute, with all the woman's vital signs stable. Following the potential abdominal trauma experienced by the client, which obstetric emergency must be excluded? A. Abruptio placentae B. Ectopic pregnancy C. Placenta previa D. Massive uterine rupture

B Choice B is correct. Raking leaves exposes the child to allergens from the trees. The nurse should advise the parents to seek an alternative activity that will allow the child to continue to help with the work at home but minimize exposure to potentially asthma attack-inducing allergens. Choice A is incorrect. Although studies vary across the board, there is no reliable, large-scale study showing the use of brass instruments (including trumpets) is harmful to those with asthma. Choice C is incorrect. Extracurricular activities are encouraged to promote maturity, develop social skills, and foster friendship among colleagues in children. Choice D is incorrect. Swimming is a suitable exercise for the lungs. Environmental allergen exposure is one of the most common causes of asthma exacerbation. Asthma triggers range from environmental allergens and respiratory irritants to infections, aspirin, exercise, emotion, and gastroesophageal reflux disease. Client education is key to avoiding asthma attack triggers.

An emergency department nurse is caring for a pediatric client who arrived experiencing an acute asthma attack. Once controlled, the nurse interviews the client's parents to determine which of the pediatric client's activities could precipitate the client's asthma attacks. Which statement by the parents would warrant the nurse to provide additional teaching? A. "Our child loves playing the trumpet in the grade school band." B. "Our child rakes leaves every Saturday afternoon to help with the work at home." C. "Our child participates in extracurricular activities." D. "Our child swims five laps twice a week with friends."

C

An emergency department nurse is taking care of a 68-year-old female after she fell. The paramedics said that she was on the bathroom floor for approximately 10 hours. The nurse is straight catheterizing the patient for a urine sample when she notices the amount of urine reaches 800 mL. The urine is still flowing heavily. What action should the nurse take and why? A. Drain the patient's bladder entirely and place a small amount in a urine specimen cup. This patient needs a urine sample to check for rhabdomyolysis. B. Continue draining the bladder fully, then place a Foley catheter to monitor for sufficient urine output. C. Stop draining the patient's bladder because the patient is at risk for developing bladder spasms. D. Stop draining the patient's bladder and consult the physician for further instructions.

C Choice C is correct. Ergonomically designed chairs are commonly designed with a primary focus on providing lumbar spine support. Although the chairs often provide some level of support to various levels of the spinal column, the lumbar spine is the most common region for back pain to occur and therefore is the spinal region ergonomically designed chairs routinely support. Each curve of the spine (including lumbar) is shown in the image below. Choice A is incorrect. The cervical spine is not typically supported by ergonomically designed chairs. Choice B is incorrect. Although ergonomically designed chairs do support the thoracic spine when an individual leans back, proper positioning would often have an individual sitting upright at a 90-degree angle. Choice D is incorrect. While ergonomically designed chairs do support the sacral spine, the chairs concentrate on the lumbar region of the back primarily due to society's high prevalence of lumbar back pain complaints. The lumbar spine—where most back pain occurs—includes five vertebrae (L1-L5). The lumbar region supports the majority of the upper body weight.

Ergonomically designed chairs are best designed to provide support to which region of the spine? A. The cervical spine B. The thoracic spine C. The lumbar spine D. The sacral spine

D

The nurse has become aware of the following client situations. The nurse should first follow up with which client? A. A client with a chest tube that has tidaling in the water seal chamber. B. A client that is receiving mechanical ventilation and is occasionally biting on the tube. C. A client that is receiving albuterol via a nebulizer and reports headache and nervousness. D. A client with pneumonia that has become restless and confused.

A Choice A is correct. Here, the nurse should hold the medication and contact the HCP to clarify the medication order, as multiple red flags have been presented in the scenario. First, subcutaneous heparin is typically used as prophylaxis against deep vein thrombosis (DVT) and pulmonary embolism (PE), with intravenous heparin used in the event of a PE developing. Here, the client has a PE, raising questions about the route of administration ordered. Second, an order for 50,000 units of heparin should raise questions, as this dose is significantly higher than any dose appropriate for a PE client. The prescribed dosage of 50,000 units is unsafe and would put the client at an increased risk for bleeding. Therefore, the nurse should hold the medication and contact the HCP to clarify the medication order. [As a footnote, following order clarification (i.e., when the medication is ultimately administered), the client should be informed that in this instance, the medication is being administered not to prevent blood clots, but to treat blood clots, as this client currently has a pulmonary embolism.] Choice B is incorrect. The nurse must never alter the prescribed dosage of any medication. If a dosage adjustment is needed, the nurse should contact the HCP, discuss the concern(s), and obtain the appropriate order from the HCP. Here, administering an amount different than that ordered would be a medication error. Moreover, knowingly administering less than the dose ordered without clarifying with the HCP places the nurse at an increased liability risk if harm to the client does subsequently occur. Choice C is incorrect. Prior to administering this dose of heparin, the nurse should verify the order. Monitoring the client for signs and symptoms of bleeding is an appropriate nursing intervention. Choice D is incorrect. Before administering this dose of heparin, the nurse should verify the order. Bleeding precautions and instructing the client to remain in bed are appropriate nursing interventions. Heparin is a "high-alert" medication. Heparin is commonly used for deep vein thrombosis (DVT) prophylaxis in a dose of 5,000 units two to three times per day given subcutaneously. When used for prophylaxis, heparin does not need to be monitored. Heparin is given by continuous intravenous infusion when used therapeutically.

The health care provider (HCP) has prescribed 50,000 units of heparin via subcutaneous injection for a client with a pulmonary embolism (PE). The vial on hand contains 20,000 units per mL. The nurse calculates that the drug volume to be administered will be 2.5 mL. The nurse verifies that the client understands the action of the medication when the client states: "This medication will help prevent blood clots." After double-checking the dosage to be administered, the nurse decides to do which of the following? A. Hold administration and contact the health care provider (HCP) to clarify the medication order B. Administer 0.2 mL of the medication instead of the calculated volume of 2.5 mL C. Administer the prescribed dose while monitoring the client for signs of bleeding D. Administer the medication as prescribed, initiate bleeding precautions, and instruct the client to remain in bed to prevent injury

A, C, D, E

The nurse is caring for a client with atrial fibrillation who takes prescribed warfarin. Which alternative therapies should the nurse advise this patient to avoid? Select all that apply. A. Ginger root B. Aloe vera C. Garlic [29%] D. Ginko biloba E. Saw palmetto

C

The nurse has attended a continuing education conference regarding medication administration and meal times. Which statement, if made by the nurse, would indicate correct understanding? A. Proton pump inhibitors (PPIs) should be given as the client eats their breakfast. B. Glucocorticoids should be given on an empty stomach to prevent gastrointestinal irritation. C. Rapid-acting insulins should be administered approximately 10-15 minutes before meals D. Levodopa-Carbidopa should be administered with a high-protein snack to enhance its absorption.

B

The nurse is caring for a client with heart failure. Which medication should the nurse clarify with the primary healthcare provider (PHCP)? A. Lisinopril B. Prednisone C. Hydralazine D. Carvedilol

B Choice B is correct. If the ICD administers shock, others in physical contact with the patient may feel it but will not be harmed. A shock from the ICD indicates that it's effectively treating the rhythm disorder. Choice A is incorrect. If the ICD administers shock, others in physical contact with the patient may feel it but will not be harmed. Choice C is incorrect. The ICD does not produce any magnetic field. Choice D is incorrect. The shock felt by the patient's friend is an expected outcome of a functional ICD that's treating the patient's rhythm. The question does not mention any clinical symptoms such as chest pain following the shock. This does not necessitate an immediate checkup by a physician. If the patient had just one shock in 24 hours and feels fine after the shock; no immediate intervention is necessary. However, if the patient reports chest pain/ chest pressure/ shortness of breath following an ICD shock or if two or more shocks are experienced within a 24 hour period, the patient should seek medical attention immediately.

The client with an implantable cardioverter defibrillator (ICD) is at the outpatient department. He is concerned about a shock that his friend felt when they were shaking hands. The nurse can discuss which of the following in response to the patient's concern? A. His friend should have an ECG taken to check if his heart rhythm was affected in anyway. B. He shook hands with his friend at the exact same time the ICD delivered a shock to restore his rhythm, and that he need not worry. C. The shock was due to the magnetic field the device emits. D. He should get urgently checked by the physician.

A, D, E

The emergency department (ED) nurse is triaging clients in the ED. It would be appropriate for the nurse to triage which client as nonurgent? A client Select all that apply. A. with a localized abscess on the right leg. B. reporting that they have chest pressure. C. with nausea, vomiting, and painful urination. D. requesting a refill of their prescribed antidepressant. E. with a single laceration to the left hand.

C

The nurse has instructed a client with type 1 diabetes mellitus about proper exercise. Which of the following client statements indicates a correct understanding of the teaching? A. "I should carry a snack rich in protein just in case I feel shaky." B. "I will not take my prescribed daily glargine insulin if I plan on exercising." C. "I can initially expect my glucose level to rise with vigorous exercise." D. "I should start my exercise near the time that my insulin peaks."

A Choice A is correct. Elderly clients do not show "typical" symptoms of pneumonia, such as fever. The nurse should watch for altered levels of consciousness or behavioral changes as these may indicate decreased oxygenation to the brain from sepsis. Therefore, the nurse should see this client first. Choice B is incorrect. The client is showing the expected signs and symptoms of influenza. This patient does not require the nurse's immediate attention. Choice C is incorrect. Tidaling in a water-seal system is expected; therefore, the nurse would not need to see this client first. Choice D is incorrect. Drainage from the nose in a patient with a sinus infection is expected.

The nurse has just finished receiving the shift report from the night nurse. Which patient should the nurse see first? A. A 90-year-old patient with pneumonitis who is getting restless but is currently afebrile. B. A 20-year-old patient with influenza who is febrile and complaining of a headache. C. A 40-year-old patient with hemothorax in the right lung who is attached to a chest drainage system that is tidaling. D. A 27-year-old with sinusitis having green drainage from his nose.

B Choice B is correct. When securing an indwelling urinary catheter for a male, it is appropriate to anchor it to the lower abdomen (with the penis pointed upward) or upper thigh. The catheter tubing should be secured to the lower abdomen or the upper thigh to prevent urethral injury. Choices A, C, and D are incorrect. These are anatomical locations are inappropriate for securing an indwelling catheter. When securing an indwelling catheter for a female, it should be anchored to the inner thigh. When securing an indwelling catheter, the following should occur - ✓ The catheter should be attached to a male's upper thigh (with the penis pointed upward) or lower abdomen. ✓ For a female, the catheter should be connected to the inner thigh. The catheter should be secured with an adhesive device. The device is typically gently removed with an alcohol swab to avoid a shearing injury to the skin.

The nurse has just inserted an indwelling urinary catheter for a male client. The nurse plans on securing the catheter to the client's A. inner thigh. B. lower abdomen. C. outer thigh. D. medial thigh.

B, D Choices B and D are correct. These two statements indicate that the patient needs further follow-up education to correct the misconceptions. The client does not need to weigh themselves daily (Choice B) as that would be applicable for CHF and not for atrial fibrillation. Considering daily weight checks in CHF is useful to detect excess fluid retention, which may precede symptoms such as shortness of breath. Wearing a mask in public is unnecessary as an infection is not the concern here (Choice D). Choices A, C, and E are incorrect. These options are wrong because these statements indicate correct understanding by the patient and do not require follow-up teaching. Atrial fibrillation is a common dysrhythmia that results in a decrease in an atrial kick. A client with atrial fibrillation is at risk for an ischemic stroke (Choice A) because of the formation of clots in the atrial appendage. Treatment for atrial fibrillation range from medications (diltiazem, amiodarone) to cardiac ablation (Choice C). Finally, the client needs to notify the PHCP if they develop dyspnea because this could be an indication of atrial fibrillation with a rapid ventricular response (RVR), which requires immediate medical attention (Choice E).

The nurse has provided education to a client with atrial fibrillation. Which of the following statements by the client would require a follow-up? Select all that apply. A. "I have an increased risk for a stroke." B. "I should weigh myself daily at the same time." C. "I may be prescribed medications such as amiodarone." D. "I should wear a mask when I am in public." E. "I should follow-up with my primary healthcare provider (PHCP) if I develop shortness of breath."

A

The nurse has provided medication instruction to a client who has been prescribed metformin. Which of the following statements, if made by the client, would indicate a correct understanding of the teaching? A. "This medication may cause me to have bloating or loose stools." B. "I will need to take my blood glucose prior to taking this medication." C. "If I eat fewer carbohydrates in a day, I should skip a dose." D. "The goal of this medication is to increase my hemoglobin A1C."

A, C

The nurse in the clinic is caring for a 10-year-old with asthma. The child uses an albuterol multi-dose inhaler before engaging in exercise. The nurse should educate the child and parents that potential side effects of this short-acting beta-2 agonist (SABA) are: Select all that apply. A. Tachycardia B. Hypotension C. Headache D. Hypoglycemia

B

The nurse is assessing a 2-year-old client with the following symptoms: excessive drooling, stridor, difficulty swallowing, and difficulty speaking. Based on these assessment findings, which condition does the nurse suspect? A. Croup B. Epiglottitis C. Laryngotracheal bronchitis D. Bronchiolitis

A Choice A is correct. The nurse should assess the client for the presence of orthostatic hypotension. Orthostatic hypotension is often seen in association with hyponatremia secondary to dehydration. Orthostatic or postural hypotension refers to a significant decrease in systolic blood pressure of greater than 20 mmHg or a reduction of at least 10 mmHg in diastolic pressure upon 3 to 5 minutes of standing. Choices B, C, and D are incorrect. Peaked T-waves are associated with hyperkalemia, not hyponatremia. Bounding peripheral pulses would be a clinical finding associated with fluid volume overload. Polyuria would be a finding associated with hyperglycemia and diabetes inspidus. This is not a finding found with hyponatremia and dehydration.

The nurse is assessing a client admitted with hyponatremia secondary to dehydration. Which of the following physical assessment findings would be expected? A. Orthostatic hypotension B. Peaked T-waves on electrocardiogram (ECG) C. Bounding peripheral pulses D. Polyuria

Stop the transfusion Administer oxygen Take vital signs Obtain a urine specimen

The nurse is caring for a patient receiving a blood transfusion. On assessment, the nurse notes that the patient's respirations are rapid, the face is flushed, and the patient is complaining of itching. The nurse suspects the patient is having a transfusion reaction. The nurse should accomplish the following actions: Take vital signs Stop the transfusion Administer oxygen Obtain a urine specimen. The nurse should complete the tasks in the following order:

C Choice C is correct. Phantom limb pain (PLP) is a form of neuropathic pain that can be treated with medications such as pregabalin, gabapentin, amitriptyline, or propranolol. Propranolol is a beta-adrenergic blocker, and while its action related to PLP is not fully understood, it has demonstrated efficacy for this type of pain. Choices A, B, and D are incorrect. Aripiprazole is an atypical antipsychotic and is not indicated in the management of PLP. Oxycodone is an opioid, and while opioids are effective for nociceptive pain, they have limited benefits for neuropathic pain. Hydroxyzine is a histamine antagonist commonly used in the treatment of allergies and anxiety. Phantom limb pain (PLP) may occur after an above-the-knee amputation. This type of neuropathic pain is often described as a burning, crushing, or cramping sensation. It is essential for the nurse to acknowledge the pain and refrain from being dismissive (for example - telling the client that your limb is no longer present). Measures to treat PLP include medications such as pregabalin, gabapentin, amitriptyline, or propranolol.

The nurse is assessing a client with phantom limb pain following an above-the-knee amputation. The nurse anticipates a prescription for A. Aripiprazole B. Oxycodone C. Propranolol D. Hydroxyzine

A, C, D, F Choices A, C, D, and F are correct. Low socioeconomic status, a history of being a sex worker, illicit drug use, and a previous history of sexually transmitted infections are all risk factors for contracting STIs. Other factors include numerous sexual partners and being unmarried. Choices B and E are incorrect. A history of cancer and exclusive/monogamous relationships are not examples of risk factors for acquiring an STI.

The nurse is assessing her prenatal client for sexually transmitted infections (STIs) by looking for risk factors. Which of the following are risks of acquiring an STI? Select all that apply. A. Low socioeconomic status B. A monogamous relationship C. A past history of working in the sex industry D. Illicit drug use E. History of cancer F. Previous history of STIs

A Choice A is correct. Administering the IV antibiotic is the top priority in a client with cystic fibrosis (CF) that develops a fever. Due to the excessively thick mucus that builds up in their bronchi and bronchioles, children with CF are incredibly susceptible to respiratory infections. A fever is an indication of infection and aggressive management is the top priority. Choice B is incorrect. Pancreatic enzymes are administered to children with CF within 30 minutes of any meal and snack. These are given to aid in digestion since the excessive, sticky mucus clogs up the pancreatic duct in these clients. This is a standard medication given every day, but is not the top priority when a child with CF develops a fever. Choice C is incorrect. Fat soluble vitamins are a daily medication for children with CF. Due to the buildup of excessive, sticky mucus in their bile duct, children with CF do not absorb fat normally. This leads to a deficiency in fat soluble vitamins, which are vitamins A, D, E, and K. This is a standard medication given every day, but is not the top priority when this client develops a fever. Choice D is incorrect. Albuterol is a bronchodilator frequently given as a nebulizer treatment to clients with CF. Although this medication might be given top priority if the client was experiencing respiratory difficulty, the question states they have developed a fever. Due to this finding, the IV antibiotics are the top priority as CF clients are very susceptible to infections.

The nurse is caring for a 16-year-old client with cystic fibrosis. The client develops a temperature of 101.2 degrees F (38.4C). Which medication does the nurse administer with top priority? A. IV antibiotic B. Pancreatic enzyme C. Fat soluble vitamin D. Albuterol

A

The nurse is caring for a 3-year-old client diagnosed with bronchitis. The mother asks the nurse what this diagnosis means. Which response most correctly explains the diagnosis of bronchitis? A. "Bronchitis occurs when an infection causes inflammation in the large airways. These include the trachea and bronchi, which are in the lower part of the respiratory tract." B. "Bronchitis occurs when an infection causes inflammation in the small airways. These include the trachea and bronchi, which are in the upper part of the respiratory tract." C. "Bronchitis occurs when an infection causes inflammation in the large airways. These include the trachea and bronchi, which are in the upper part of the respiratory tract." D. "Bronchitis occurs when an infection causes inflammation in the small airways. These include the trachea and bronchi, which are in the lower part of the respiratory tract."

A, C Choices A and C are correct. Hemarthrosis is defined as bleeding into a joint cavity. Most commonly affected joints include knees, ankles, and elbows. Hemarthrosis is a frequent complication of hemophilia because of the deficiency of clotting factors and prolonged clotting times. When the nurse has a patient with hemarthrosis, she can expect joint pain and swelling, and external bruising in the hemarthrosis area due to the accumulation of blood in that joint cavity. Choice B is incorrect. A decreased level of consciousness (LOC) is not a finding expected with hemarthrosis. Hemarthrosis is defined as bleeding into a joint cavity, which would not cause a decreased LOC. Decreased LOC may be seen in patients with hemophilia if they develop a brain bleed. Other symptoms to look out for if a brain bleed is expected include slurred speech, vision changes, and headaches. Choice D is incorrect. Melena is not a finding expected with hemarthrosis. Hemarthrosis is defined as bleeding into a joint cavity, which would not cause melena. Melena is characterized by black, tarry stools and is due to upper gastrointestinal bleed. Melena is a symptom of hemophilia as well but is not associated with hemarthrosis.

The nurse is caring for a 5-year-old girl diagnosed with hemophilia with a recurrent episode of hemarthrosis. Which of the following would the nurse expect on their assessment? Select all that apply. A. Joint pain and swelling B. Decreased level of consciousness C. Bruising D. Melena

Perform hand hygiene, identify the patient, explain the procedure to the patient, and prepare the insertion kit using sterile gloves. Spread the labia and hold them open. Cleanse the meatus from front to back on the right side, then left side, and down the center. Insert the catheter and inflate the balloon. Secure the catheter to the patient, then initial the securement device with the date and time.

The nurse is caring for a female client who is incontinent of urine. The MD orders an indwelling Foley catheter to be placed. Place the following actions in the correct order for the nurse to appropriately insert the Foley catheter: Perform hand hygiene, identify the patient, explain the procedure to the patient, and prepare the insertion kit using sterile gloves. Cleanse the meatus from front to back on the right side, then left side, and down the center. Spread the labia and hold them open. Insert the catheter and inflate the balloon. Secure the catheter to the patient, then initial the securement device with the date and time.

D Choice D is correct. A client with severe pre-eclampsia should be monitored closely for seizures which are the hallmark manifestation of eclampsia. The nurse should plan care involving seizure precautions at the bedside, including suction equipment, padded side rails, and oxygen. Choices A, B, and C are incorrect. Sterile gloves, portable ultrasound, and a liter of 0.9% saline would not be necessary to manage a client having a seizure directly related to pre-eclampsia. These tools would be helpful for other obstetric procedures but not for a severely pre-eclamptic client at risk of having a seizure. Severe pre-eclampsia may require intensive care monitoring, depending on other factors. Nursing care for a client with severe pre-eclampsia includes: ➢ Appropriate safety equipment at the bedside, which includes seizure precautions. ➢ Frequent vital signs and blood pressure must be closely monitored as a hypertensive emergency may develop. ➢ Prescribed medication administration such as magnesium sulfate. ➢ Frequent fetal well-being assessment that includes continuous fetal heart rate monitoring.

The nurse is caring for a client admitted with severe pre-eclampsia. It would be essential for the nurse to have which of the following items at the bedside? A. One liter of 0.9% saline B. Sterile gloves C. Portable ultrasound D. Suction equipment

B

The nurse is caring for a client experiencing an adrenal crisis (Addisonian crisis). The nurse should be prepared to administer which intravenous fluid? A. Lactated Ringers (LR) B. 0.9% saline C. Dextrose 5% in water (D5W) D. Dextrose 5% in water and Lactated Ringers (D5LR)

A, B, C, E

The nurse is caring for a client immediately following an ultrasound-guided thoracentesis. Which client finding requires follow-up? Select all that apply. A. Nagging cough B. Trachea slanted more to the unaffected side C. Rapid heart rate D. Localized discomfort at the needle site E. Crackling sound made at the insertion site when palpated

B Choice B is correct. Tenofovir-emtricitabine is a medication used as pre-exposure prophylaxis (PrEP) for clients at high risk for HIV infection. This medication is taken daily and may provide up to 96% efficacy against HIV infections. Choices A, C, and D are incorrect. The other options are not utilized in PrEP. Voriconazole is an antifungal agent. Raloxifene is an estrogen modulator utilized in the management of breast cancer. Lurasidone is an atypical antipsychotic indicated for psychotic and mood disorders. ADDITIONAL INFO ✓ Pre-exposure prophylaxis (PrEP) is an effective medication in reducing HIV infections for those at risk. ✓ PrEP can be administered via a daily pill or an injection every two months. ✓ The efficacy of PrEP is up to 99% when taken as prescribed. ✓ Indications for a client to start PrEP include unprotected intercourse, multiple sexual partners, and intravenous drug use.

The nurse is caring for a client interested in pre-exposure prophylaxis for human immunodeficiency virus (HIV). Which prescription would the nurse anticipate? A. Voriconazole B. Tenofovir-emtricitabine C. Raloxifene D. Lurasidone

D Choice D is correct. The physical needs of the client with a mental health disorder prioritize over psychosocial needs. The client experiencing dizziness is highly concerning because this could be suggestive of severe dehydration or other electrolyte imbalances. Choices A, B, and C are incorrect. Dental caries, sores in the oral mucosa, electrolyte disturbances, dehydration, irregular menses, and calluses on the fingers are all manifestations associated with bulimia nervosa. A client expressing self-negating statements requires follow-up but does not prioritize over the client endorsing dizziness. ADDITIONAL INFO Maslow's Hierarchy of Needs signifies that physical needs must be assessed and cared for first before psychological needs can be satisfied. Thus, the priority is to take care of the client's physical need of dizziness as this is a manifestation associated with significant dehydration.

The nurse is caring for a client newly admitted to the mental health unit with bulimia nervosa. Which client statement requires immediate follow-up? A. "These sores in my mouth hurt." B. "When can I weigh myself?" C. "I hate my life and wish it was over." D. "I feel really dizzy right now."

B

The nurse is caring for a client receiving a continuous infusion of isotonic fluids and observes infiltration at the vascular access device. The nurse should take which action? A. Reduce the infusion rate and elevate the affected extremity. B. Stop the infusion and remove the intravenous (IV) catheter. C. Stop the infusion and reposition the intravenous (IV) catheter into the vein. D. Reduce the infusion rate and apply a warm compress to the intravenous (IV) site.

B

The nurse is caring for a client who appears to be developing heart failure (HF). Which of the following laboratory tests would the nurse expect the primary health care provider (PHCP) to prescribe to confirm the diagnosis? A. Basic metabolic panel (BMP) B. B-type natriuretic peptide (BNP) C. Lipid profile D. Troponin

A

The nurse is caring for a client who has polycystic kidney disease (PKD). Which of the following would indicate the client is achieving treatment goals? A. Blood Pressure 128/63 mmHg B. Creatinine 2.3 mg/dL C. Proteinuria 2+ D. Sodium 132 mEq/L

C

The nurse is caring for a client who is newly prescribed cimetidine. The nurse understands that this medication is prescribed to treat which condition? A. Cystic fibrosis B. Clostridium difficile C. H. pylori D. Crohn's disease

A

The nurse is caring for a client who is receiving prescribed cilostazol. Which of the following client findings would indicate a therapeutic response? A. Absence of pain while ambulating B. Decreased total cholesterol C. Increased visual acuity D. Improved focus and attention

D Choice D is correct. Amoxicillin is an antibiotic that is commonly used to treat Helicobacter pylori infections. When treating this infection, this medication is often coupled with a proton pump inhibitor such as esomeprazole. Choices A, B, and C are incorrect. Dicyclomine is an antispasmodic medication used to treat gastrointestinal spasms, which are common in individuals with irritable bowel syndrome. Metoclopramide is a medication that causes gastric emptying and is used for nausea and vomiting. Valacyclovir is an antiviral indicated for herpes infections. H. pylori is a gram-negative bacterium spread via the oral-oral route or oral-fecal. This bacterium may cause an individual to develop a gastrointestinal ulcer. Antibiotic therapy consisting of one or two antibiotics is often the treatment.

The nurse is caring for a client with Helicobacter pylori. The nurse should anticipate a prescription for which of the following medications? A. Dicyclomine B. Metoclopramide C. Valacyclovir D. Amoxicillin

D Choice D is correct. When evacuating from an internal disaster, the nurse should first evacuate the most ambulatory client. The client with acute glomerulonephritis only has one device, and the nurse can quickly change the system to a leg bag or instruct the client to keep the bag below their bladder. Choices A, B, and C are incorrect. The client with a below-the-knee amputation will require significant resources to mobilize. Further, the client's PCA device must be secured before evacuation. The client receiving mechanical ventilation will require manual ventilation and oxygen. Thus, requiring a significant number of resources. Finally, the client with dementia receiving enteral feedings and IV fluids must have their devices clamped and locked before evacuation. This client also is unlikely to effectively comprehend evacuation instructions and should be supervised. For a fire, the nurse should first evacuate the client(s) closest to the fire. Then, the nurse should evacuate the clients that are the most ambulatory. These individuals may assist others with ambulation out of the facility.

The nurse is caring for assigned clients. Which client should be evacuated first during a fire? A client with A. below-the-knee amputation receiving patient-controlled analgesia. B. acute respiratory distress syndrome receiving mechanical ventilation. C. advanced dementia receiving enteral feedings and intravenous fluids. D. acute glomerulonephritis with an indwelling urinary catheter.

B Choice B is correct. This is an optimal hemoglobin A1C as it is less than 5.7%. A hemoglobin A1C of 5.7% to 6.4% is prediabetes. This is a concerning finding as the client is on a negative trajectory toward diabetes mellitus. A hemoglobin A1C of 6.5% is the diagnosis of diabetes mellitus. Choices A, C, and D are incorrect. Total cholesterol of 215 mg/dL is a concerning finding. The goal is to have total cholesterol of less than 200 mg/dL. Elevated total cholesterol contributes to metabolic syndrome, which is the driver of diabetes mellitus. Fasting blood glucose of 128 mg/dL is elevated (this is impaired fasting glucose), and a level greater than 126 mg/dL requires further testing for diabetes mellitus. Random blood glucose of 210 mg/dL is concerning as this is a provisional diagnosis for diabetes mellitus. ADDITIONAL INFO The following are diagnostic criteria for diabetes mellitus A1C >6.5%. The test should be performed in a laboratory using an NGSP-certified method and standardized to the DCCT assay. AND Fasting blood glucose greater than or equal to 126 mg/dL (7.0 mmol/L). Fasting is defined as no caloric intake for at least 8 hours. OR During oral glucose tolerance testing, two-hour blood glucose equals or exceeds 200 mg/dL. The test should be performed using a glucose load containing the equivalent of 75 g of anhydrous glucose dissolved in water. OR A casual or random blood glucose concentration greater than 200 mg/dL in a client with classic manifestations of hyperglycemia or hyperglycemic crisis. Casual is defined as any time of the day without regard to time since the last meal. The classic symptoms of diabetes include polyuria, polydipsia, and unexplained weight loss.

The nurse is counseling a client who has prediabetes. The nurse understands that the client is meeting the treatment goal as evidenced by A. total cholesterol of 215 mg/dL. B. hemoglobin A1C of 5.4%. C. fasting blood glucose 128 mg/dL. D. random blood glucose of 210 mg/dL.

B, C, E

The nurse is developing a plan of care for a client diagnosed with Addison's disease. Which of the following should the nurse include in the client's plan of care? Select all that apply. A. Diet high in potassium B. Continuous telemetry monitoring C. Intravenous hydrocortisone D. Fluid restriction E. Fall precautions F. Indwelling urinary catheter

A, B, C

The nurse is educating a diabetic client regarding foot care. Which of the following statements by the client indicates that he understood the nurse's instructions? Select all that apply. A. "I need to check my feet daily for sores, blisters, dry skin, and cuts." B. "I need to wash my feet daily and keep them dry." C. "If I get sores or blisters on my feet, I should not pop them." D. "I need to apply cream to my heels and between my toes daily

A, B, E

The nurse is educating a group of students on the effects of corticosteroids. It would be appropriate for the nurse to identify the following adverse effects associated with corticosteroids. Select all that apply. A. Mood lability B. Immunosuppression C. Hypoglycemia D. Hyperkalemia E. Weight gain

A, B, E

The nurse is evaluating a client taking levothyroxine for hypothyroidism. Which findings indicate that the client is experiencing an adverse effect? Select all that apply. A. Heat intolerance B. Palpitations C. Bradycardia D. Constipation E. Insomnia F. Weight gain

D

The nurse is giving discharge instructions to a client recently diagnosed with vaginitis. Which of the following instructions should the nurse include? A. Use oral contraceptives during sexual intercourse. B. Practice regular douching. C. Abstain from eating yogurt. D. Wear loose-fitting clothing and cotton underwear.

D

The nurse is going over the list of assigned clients for the shift. The nurse knows which client is most at risk for experiencing a fluid volume deficit? A. A client with cirrhosis B. A client with an ileostomy and normal amount of output C. A client with a BUN of 32 and creatinine of 2.7 D. A client with diabetes insipidus and an NG tube set to low intermittent wall suction

A Choice A is correct. Signs and symptoms of cervical cancer include back and leg pain, spotting between menstrual periods and after intercourse, vaginal discharge, and lengthening of a menstrual period. A pap smear is needed to assess cellular changes (i.e. check for cancerous and precancerous conditions). Choice B is incorrect. Endometrial cancer manifests as menorrhagia (excessive menstrual bleeding), low abdominal pain, backache, and constipation due to pressure from an enlarging mass. A biopsy is needed to confirm the diagnosis. Choice C is incorrect. Initial signs and symptoms of ovarian cancer include the following: an increasing abdominal girth due to ovarian enlargement, constipation due to rectal pressure from the enlarging mass, anemia, vomiting, and cachexia. Choice D is incorrect. A bacterial infection causes vaginitis. Signs and symptoms include pruritus, burning urination, dysuria, dyspareunia, and a foul-smelling vaginal discharge.

The nurse is in the screening room of a women's health clinic. The nurse notices a particular woman complaining of back and leg pain, spotting after intercourse with her husband, and vaginal discharge for the past few months. The nurse suspects: A. Cervical cancer B. Endometrial cancer C. Ovarian cancer D. Vaginitis

A Choice A is correct. DKA treatment aims to lower the blood glucose by 50 to 75 mg/dL/hr. This is accomplished by the prescribed regular insulin, which is given intravenously. Choices B, C, and D are incorrect. Dextrose 50% should be available in the event of severe hypoglycemia. Dextrose 5% is not sufficient to treat hypoglycemia. The treatment goal for the hypovolemia caused by DKA is isotonic saline, not hypertonic saline. Urine output would decrease with the infusion of regular insulin as correcting the hyperglycemia would treat the polyuria, which is a symptom of hyperglycemia. DKA is a common complication associated with type one diabetes mellitus. DKA may cause both hyperglycemia and hypovolemia. Treatment for hyperglycemia includes the initiation of a prescribed regular insulin bolus at 0.1 unit/kg followed by a continuous infusion at 0.1 unit/kg/hr. This treatment will also target the acidosis found with DKA. Treatment for hypovolemia includes the initiation of isotonic saline infusions at 15 to 20 ml/kg during the first hour. Fluid volume treatment has been effective when urine output restores to normal, the heart rate is within normal limits, and the client is normotensive. For the client receiving regular insulin, close monitoring of their blood glucose and potassium is essential. The most common complications of regular insulin therapy are hypoglycemia and hypokalemia.

The nurse is planning a staff development conference about diabetic ketoacidosis (DKA). Which of the following information should the nurse include? A. The goal is to lower blood glucose by 50 to 75 mg/dL/hr. B. Dextrose 5% should be available for hypoglycemia symptoms. C. Hypovolemia caused by DKA may be treated with 3% saline. D. The urine output would increase once regular insulin is initiated.

B Choice B is correct. Homonymous hemianopia (HH) is vision loss on the same side of the visual field in both eyes. It is appropriate for the nurse to teach the client to scan the room. Scanning the room will expand the visual field because the same half of each eye is affected. Choices A, C, and D are incorrect. An eye patch is an appropriate intervention for a client with double vision (diplopia). HH is not a problem with hearing and changing the approach to speaking to a client and providing ear plugs is irrelevant to this disorder. Homonymous Hemianopia is characterized by vision loss on the same side of the visual field in both eyes. This is usually caused by a stroke, tumors, or epilepsy. Visual field loss is indicative of a lesion involving the visual pathway posterior to the chiasm.

The nurse is planning care for a client with homonymous hemianopia. The nurse should plan for which intervention in the care plan? A. Place an eye patch over the affected eye B. Instruct the client to turn their head from side to side C. Speak slowly, clearly, and in a deeper voice D. Provide the client with ear plugs to promote rest

C Choice C is correct. The nurse should make sure that the patient receives a prescribed bronchodilator about 15 minutes before their chest physiotherapy procedure. Chest physiotherapy is used to loosen secretions trapped in the lungs. When administered before this procedure, a bronchodilator helps to dilate the bronchioles and liquify secretions. Choice A is incorrect. A gown or piece of fabric should be placed between the hands or percussion device right before the procedure. However, this should be done just before the process. Another option (administering bronchodilator 15 minutes prior) exists in the choices and is the initial action. Choice B is incorrect. Walking with the patient before the procedure is not necessary before chest physiotherapy. Choice D is incorrect. Calling the physician to confirm the x-ray results is not necessary at this time and does not alter the plan for chest physiotherapy.

The nurse is planning to assist a respiratory therapist in performing a chest physiotherapy procedure. Which of the following is the initial action by the nurse before the process? A. Place a gown or fabric between the hands or percussion device and the client's skin B. Walk with the patient for a few laps around the unit to aid in percussion C. Administer a prescribed bronchodilator D. Call the physician to confirm x-ray results

D

The nurse is preparing a presentation on Cushing's disease. It would be correct if the nurse states that Cushing's disease is caused by A. destruction to pancreatic beta cells. B. excessive discharge of thyroid-stimulating hormone (TSH). C. decrease in the secretion of androgens and glucocorticoids. D. increase in the secretion of adrenocorticotropin hormone (ACTH).

A, B, C, E

The nurse is preparing to obtain capillary blood glucose (CBG) for a client with diabetes mellitus. The nurse should take which action? Select all that apply. A. Apply gloves for this procedure B. Clean the finger with isopropyl alcohol C. Collect the second blood drop on the test strip D. Prick the central part of the finger for the sample E. Clean and disinfect the glucometer in between uses

A, B, C Choices A, B, and C are correct. A pulse oximetry device should be provided to the client, and they should be encouraged to log their oxygen saturations as directed. If the client experiences dyspnea or tachypnea, the client should be instructed to seek medical attention for a level less than 95% (unless otherwise directed). Padding the tubing around pressure ears (back of the ears) is recommended to avoid injury. A sign posted on the door should be visible to alert visitors of the oxygen and extinguish and open flames. Choices D and E are incorrect. Stovetop and oven cooking is highly discouraged as the presence of oxygen may accelerate any fire that may ignite. Rather, if cooking is to be done using heat or flames, another individual should do the cooking, and the oxygen should be greater than six feet away from the flame source. Petroleum jelly should not lubricate the nares as it may be aspirated. Water-soluble jelly is recommended. For a client being discharged with oxygen therapy, important teaching points to emphasize include: Have a pulse oximetry device readily available. Avoiding any open flame or heat. This includes an oven, stovetop, candles, matches, and cigarettes. Flammable products such as alcohol and oil should be avoided. Have working smoke detectors in the home as well as fire extinguishers. Use a water-soluble jelly to lubricate the nasal passages and mouth to prevent drying.

The nurse is providing discharge instructions to a client prescribed nasal cannula oxygen. Which of the following instructions should the nurse include? Select all that apply. A. Keep a pulse oximetry device readily available. B. Pad the tubing in areas that put pressure on the skin. C. Have a sign on your door indicating the presence of oxygen. D. Use the oven and not the stovetop to cook. E. You may apply petroleum jelly to your nares to prevent drying.

B

The nurse is providing discharge instructions to a client prescribed phenazopyridine. Which of the following instructions should the nurse include? A. The amount of urine you void will increase B. Your urine will turn orange in color C. You may notice that your urine is malodorous D. Concentrated urine is an expected finding

D Choice D is correct. A shower shield should be placed over the tracheostomy when the client bathes. This would prevent water from entering the tracheostomy and potentially lead to pneumonia. Choices A, B, and C are incorrect. Lemon glycerin swabs are not recommended for mouth care because they are drying and may increase the number of bacteria in the mouth. This may lead to dental caries or pneumonia. Removing the old tracheostomy ties before applying the new ones is dangerous because this may allow decannulation if the client coughs. If the inner cannula is not disposable, it should not be washed with tap water. Rather, ½ strength hydrogen peroxide and sterile saline should be used to clean the device. Home care measures for a tracheostomy should be thoroughly taught to the client and the caregiver. These topics should include tracheostomy care, emergency procedures, and oxygen safety. A shower shield is utilized while the client is bathing and comes in various materials. The nurse should emphasize using this shield to decrease the risk of pneumonia.

The nurse is providing discharge instructions to a client with a tracheostomy. Which of the following instructions should the nurse include? A. You may use lemon glycerin swabs for mouth care. B. Remove the old tracheostomy ties before applying the new ties. C. You may use warm tap water to clean the inner cannula. D. Wear a shower shield over the tracheostomy when bathing.

A, B Choices A and B are correct. A Calcium of 7.9 mg/dL is critically low (normal 9.0 - 10.5 mg/dL) and requires the nurse to follow up with the PHCP. A potassium level of 3.3 mEq/dl is low (normal 3.5 - 5.0 mEq/dL), and the PHCP should also be notified of this finding. Choices C, D, and E are incorrect. The laboratory values for the sodium (normal 135-145 mEq/dL), BUN (normal 10-20 mg/dL), and creatinine (0.6-1.2 mg/dL for males) are all within normal limits and do not require notification to the PHCP.

The nurse is reviewing laboratory data for a male client scheduled for surgery. Which laboratory data requires follow-up with the primary healthcare physician (PHCP)? Select all that apply. A. Calcium 7.9 mg/dL B. Potassium 3.3 mEq/L C. Sodium 143 mEq/L D. BUN 17 mg/dL E. Creatinine 0.9 mg/dL

A

The nurse is supervising a student nurse administer prescribed medications via a double-lumen nasogastric tube (NGT) with an air vent. Which action by the student requires follow-up? The student A. Irrigates the air vent before medication administration with water B. Contacts the pharmacy to obtain available medications in liquid form C. Flushes the NGT between medications with water D. Administers each medication separately through the NGT

B Choice B is correct. The client is in obvious respiratory distress. The nurse needs help with initiating life-saving procedures such as endotracheal intubation. The nurse need not call a "Code Blue" since the client is still breathing. However, a Rapid Response Team (RRT) can be called for help. The RRT is a team of healthcare professionals who respond to client emergencies even when they are still breathing and/or have a pulse. Since the client is in obvious respiratory distress, no additional assessment is needed prior to calling the RRT. Choice A is incorrect. The client is in obvious respiratory distress, even without the oxygen saturation reading. The nurse should initiate a nursing intervention to help the client. RRT needs to be contacted right away. Please note, "When in distress do not assess!" Choice C is incorrect. The Trendelenburg position is appropriate for clients in shock, but it is inappropriate for clients in respiratory distress. Choice D is incorrect. The client is already in respiratory distress. Checking the dressing of the client is inappropriate and causes an unnecessary delay of priority interventions.

The nurse is taking care of a client two days post lobectomy. He is complaining of difficulty breathing. He is restless, lethargic, and has bilateral crackles. What is the nurse's most appropriate initial intervention? A. Check the client's oxygen saturation. B. Notify the rapid response team (RRT). C. Place the client in Trendelenburg position. D. Check the client's surgical dressing.

A, D

The nurse is teaching a client about congestive heart failure (CHF). Which of the following information should the nurse include? Select all that apply. A. "Foods such as canned vegetables and luncheon meat should be avoided." B. "Weigh yourself daily and notify the physician when weight gain is more than ten pounds in a week." C. "You may continue to take ibuprofen for your aches and pains." D. "Annual immunizations such as the influenza vaccine are recommended." E. "If you feel sick, you will need to check your urine for ketones."

A, C, E

The nurse is teaching a group of students about incident reports. Which of the following statements, if made by the student, would require further teaching? Select all that apply. A. "Reporting can only be completed if it is within one hour after the event." B. "Witnesses to an incident should be mentioned in the report." C. "A client eloping does not require an incident report." D. "A slip and fall by a client should be reported." E. "Incidents involving visitors do not have to be reported."

A, C, E

The nurse is teaching a group of students about renal disorders. Which statement, if made by the student, requires follow-up? Select all that apply. A. "Pyelonephritis causes a client to have massive amounts of proteinuria." B. "Acute kidney injury may be caused by nephrotoxic medications." C. "Bacterial cystitis is diagnosed using a 24-hour urine collection." D. "Polycystic kidney disease may cause hematuria after a cyst rupture." E. "Diabetic nephropathy is prevented by increasing the hemoglobin A1C."

D

The nurse is teaching a group of students the causes of metabolic alkalosis. It would indicate a correct understanding of the student to state which condition causes this acid-base imbalance? A. Hyperventilation B. Urinary retention C. Opioid toxicity D. Excessive vomiting

B, D

The nurse is teaching a patient who is scheduled for a thoracentesis. Which of the following information should the nurse include? Select all that apply. A. "This procedure will require you to receive general anesthesia." B. "You will need to report any shortness of breath following the procedure." C. "You will need to empty your bladder before this procedure." D. "After the procedure, a follow-up chest x-ray will be done." E. "You will need to be on a clear liquid diet one day before the procedure."

B, E Choices B and E are correct. A nonstress test is performed in the third trimester if the client has indications such as a high-risk pregnancy that may result in a stillbirth or complications such as fetal hypoxia. Ultrasounds typically require a full bladder as the fluid moves the uterus upward and assists with visualization. Choices A, C, and D are incorrect. Oral glucose tolerance testing is completed between 24-28 weeks of gestation. This test is used to determine if the client has gestational diabetes and does not take into account fetal activity. Amniocentesis is an antepartum test that may be used to determine the gender of the fetus, lung maturity, neural tube defects, or chromosomal abnormalities. Chorionic villus sampling is a test that may be performed as early as ten gestational weeks to determine if the fetus has any chromosomal abnormalities. Amniocentesis is performed for a variety of indications and at different gestational ages. Common indications for amniocentesis include identifying chromosomal, metabolic, or genetic abnormalities. Amniocentesis can assist in determining fetal lung maturity (FLM) status. After this procedure, women should be instructed to report signs of bleeding, amniotic fluid that continues to leak after 24 hours, severe cramping that lasts several hours, or a temperature greater than 100.4°F

The nurse is teaching parents about antepartum testing. Which statements should the nurse include? Select all that apply. A. "Oral glucose tolerance testing will measure fetal activity at certain intervals." B. "A nonstress test may be used to measure fetal heart rate." C. "Amniocentesis may be used to assess if you have preeclampsia." D. "Chorionic villus sampling may be done to assess for neural tube defects." E. "You may need to fill up your bladder prior to an ultrasound."

A, C, D, E

The nurse is teaching progressive relaxation techniques to a client. Which of the following statements by the client indicates that the teaching has been effective? Select all that apply. A. "I will breathe in and out in rhythm." B. "I expect my pulse to be faster afterwards." C. "I expect to require less pain medication." D. "I expect my muscles to feel less tense." E. "I will report any increased sensitivity."

C, E Choices C and E are correct. These actions by the student are incorrect and require follow-up by the nurse. When cleaning the labia with the antiseptic solution, the student should wear sterile gloves to clean the labia with their dominant hand while separating the labia with the fingers of the nondominant hand (now contaminated) to fully expose the urethral meatus. Acting correctly will greatly decrease the risk of contamination. The drainage bag of the urinary catheter should not be secured to the bed's side rails as it will move and cause tension on the tubing that may cause urinary trauma. Choices A, B, and D are incorrect. These actions by the student are correct and do not require follow-up by the nurse. Clean gloves are worn to clean the perineal area with soap and water (sterile gloves are used to apply the povidone iodine or other antiseptic solution). Asking the client to bear down gently and slowly insert the catheter through the urethral meatus is appropriate because it relaxes the external urinary sphincter, which facilitates the passage of the tubing. The urinary catheter tubing should be secured to the inner thigh of a female. The device is then secured to the bed's frame below the bladder. ADDITIONAL INFO ✓ An RN or LPN may insert, manage, and remove an indwelling urinary catheter. ✓ A UAP may assist with gathering supplies, sending appropriate specimens to the lab, and recording urinary output from the device. ✓ Pretesting the indwelling catheter balloon by injecting fluid from the saline syringe is no longer recommended. This distorts the balloon and may cause urinary tract trauma during the insertion. ✓ When securing the tubing for an indwelling urinary catheter for a male, it is secured to the upper thigh or lower abdomen (with the penis directed toward the chest). For a female, it is secured to the inner thigh. ✓ It is critical to stop the procedure for any contamination and restart the procedure with new supplies.

The nurse observes a student inserting an indwelling urinary catheter into a female client. Which action by the student requires follow-up by the nurse? Select all that apply. The student A. applies clean gloves to clean the perineal area with soap and water. B. asks the client to bear down gently and slowly insert the catheter through the urethral meatus. C. separates the labia with the fingers of the dominant hand when cleaning with antiseptic solution. D. secures the catheter tubing to the inner thigh. E. attaches the drainage bag to the side rails of bed.

A

The nurse prepares a client for a scheduled percutaneous coronary intervention (PCI). Which client statement should be reported to the primary healthcare provider (PHCP)? A. "I took my metformin this morning." B. "I get anxious when I am in closed spaces." C. "I am allergic to shellfish." D. "I may feel a warm sensation during the procedure."

A

The nurse receives a prescription for sevelamer. The nurse plans on administering this medication A. with the client's meals. B. immediately before hemodialysis. C. with a prescribed proton pump inhibitor (PPI). D. right before the client goes to bed.

A, B, D Choices A, B, and D are correct. When a child has a cleft lip and palate, the tissue and bone inside their mouth are not appropriately fused, meaning there is a space between their upper lip and palate. Ear infections will be a frequent complication for these patients due to the eustachian tube dysfunction, which connects the middle ear and the throat. Feeding issues are a common complication of cleft lip and cleft palate because it is harder for these infants to eat with the abnormality in their palate. The space in the roof of the mouth makes it very hard to suck and make a good seal around the bottle or nipple. Speech and language delays are common complications of cleft lip and palate. The roof of the mouth and lip have spaces that decrease muscle function and lead to delayed or abnormal speech. Eventually, many of these patients will require consultation with a speech-language pathologist. Choices C and E are incorrect. Weight gain is not a common complication of cleft lip and cleft palate. With these abnormalities, it is much more difficult for the infant to eat, and they commonly experience feeding issues— leading to weight loss or failure to thrive, not weight gain. Esophageal reflux is not consistent with this condition and is associated with gastroesophageal reflux disease (GERD).

The nurse reviews cleft lip and cleft palate with a group of students. It would indicate effective teaching if the student states which are the following complications of both? Select all that apply. A. Ear infections B. Feeding difficulties C. Weight gain D. Speech delay E. Esophageal reflux

A

Which aspects of the HEALTH belief model can you incorporate into your practice in a primary healthcare environment to maintain physical, mental, and spiritual health? A. The promotion of a healthy diet, social support systems, and religion B. The wearing of symbolic clothing, relaxation, and religious rituals C. The consumption of special foods, relaxation, and religious rituals D. The use of curanderos, massage, and meditation

B, C, D, E Choices B, C, D, and E are correct. This device is an external fixator and has pins directly into the skull. Any signs or symptoms of infection should be reported immediately, as these manifestations may suggest osteomyelitis. The wrench of the device should always be kept affixed to the front of the vest in the case of the need to perform CPR. It would be correct for the client to get out of bed by pushing against the mattress. Cotton clothing should be worn under the vest to absorb any excess moisture and prevent skin breakdown. Choice A is incorrect. These statements are incorrect. Driving a vehicle and riding a bicycle are prohibited while in this device because of the client's inability to turn from side to side to view traffic. The halo fixator device may be connected to traction or a vest. This device is used for cervical spinal cord fractures. The client will need to meet with occupational and speech therapists prior to discharge. The client should be instructed to report any loosening at the pin sites or foul-smelling drainage. The wrench to the device should be taped to the front of the device. This is to be used to take down the front two bars to perform CPR. The client should use a straw for liquids. The client is not permitted to ride a bike or drive a car. The client should be instructed to wear cotton t-shirts under the vest to absorb excess moisture. The client should not use any lotion or powder under the vest. Sponge baths should be used during this time. The client should wear sturdy footwear. No heels.

The nurse teaches a client about their newly applied halo fixator device with a vest. Which of the following statements should the nurse make? Select all that apply. A. "You should ride a bicycle instead of driving a car." B. "Report any fever or drainage at the pin sites." C. "Always keep the wrench taped to the front of the vest." D. "When you are getting out of bed, roll to the side and push on the mattress." E. "Wear a cotton t-shirt under the vest to absorb any moisture."

B

The patient is diagnosed with acute kidney failure. Which of the following is an appropriate psychosocial problem for the RN to include in the care plan? A. Imbalanced nutrition: less than body requirements related to altered metabolic state and dietary restrictions. B. Anxiety related to the disease process and uncertainty of prognosis. C. Excess fluid volume related to compromised regulatory mechanisms secondary to acute renal failure. D. Risk for infection related to invasive procedures and an altered immune response secondary to renal failure.

D Choice D is correct. The first specimen is discarded because it is considered "old urine" or urine in the bladder before the test began. A 24-hour urine collection helps diagnose kidney problems. It is often done to see how much creatinine clears through the kidneys. It's also done to measure protein, hormones, minerals, and other chemical compounds. Proper education on the collection of the 24-hour specimen is essential, as retaining the first specimen can cause an error in the result. Choices A, B, and C are incorrect. After the first discarded specimen, urine is collected for 24 hours.

The physician has ordered a 24-hour urine specimen. After explaining the procedure to the client, the nurse collects the first specimen. This specimen is then: A. Placed in a separate container and later added to the collection. B. Saved as part of the 24-hour collection. C. Tested and then discarded. D. Discarded and then the collection begins.

A

This nurse is caring for a client who is receiving prescribed hydralazine. Which of the following findings would indicate a therapeutic response? A. Blood pressure 130/70 mm Hg B. Pulse (P) 67/minute C. Total cholesterol 185 mg/dL D. aPTT 45 seconds

A Choice A is correct. Aplastic anemia leads to pancytopenia, a severe decrease in all hematological cell types: red blood cells, white blood cells, and platelets. Aplastic anemia may be caused by primary bone marrow failure or from secondary causes such as medications. Some medications that cause aplastic anemia include chloramphenicol, phenylbutazone, sulfonamides, anticonvulsants, cimetidine, and NSAIDs. Drug-induced aplastic anemia is the result of an idiosyncratic hypersensitivity reaction and is often reversible. In such drug-related aplastic anemias, the nurse must notify the physician and withdraw the offending agent. Choice B is incorrect. Leucocytosis refers to increased white blood cells. Leucocytosis can be induced by some medications (for example, Lithium); however, it is not serious bone marrow toxicity. Instead, Leukopenia (a reduced number of white blood cells) is more serious and makes the patient susceptible to infection. Choice C is incorrect. Thrombocytosis refers to increased platelet count. It is not the most serious form of bone marrow toxicity. Severe thrombocytopenia (a reduced number of platelets) is more serious as it causes the blood not to clot as easily and increases bleeding risk. Choice D is incorrect. A granulocyte is a type of white blood cell. White blood cells are classified into two types: granulocytes (neutrophils, eosinophils, basophils) and agranulocytes (monocytes, lymphocytes). Granulocytosis is an elevated granulocyte count. Granulocytosis may be seen as the body responds to an infection. However, it is not as serious as agranulocytosis. Agranulocytosis (granulocytopenia) is a more serious disorder that causes a severe decrease in the neutrophil count and predisposes the client to severe infections. Medications that cause agranulocytosis include antithyroid medications (carbimazole and methimazole); anti-inflammatory medications (sulfasalazine, nonsteroidal anti-inflammatory drugs), and some antipsychotics (clozapine).

While reviewing medication-related hematological side effects, the nurse recognizes which of the following as the most severe form of bone marrow toxicity: A. Aplastic anemia B. Leukocytosis C. Thrombocytosis D. Granulocytosis

B, C Choices B and C are correct. It is important to teach parents that electrical outlets should have plug covers in place by the time their child is 7-months-old. At this age, the infant will be able to crawl and will be reaching out to touch unfamiliar things. This is when electrical outlets start to pose a risk and should therefore be covered (Choice B). It is important to teach parents that toilet lids should have locks on them by the time their child is 7 months old. At this age, the infant will be able to crawl and will be reaching out and pulling themselves up on things. This is when toilets start to pose a risk and should have locks placed on their lids so that they cannot fall into them (Choice C). Choice A is incorrect. It is not necessary to use stair gates until the child is in kindergarten, as they should be able to safely navigate stairs on their own by 3 to 4 years of age. Advise parents to use stair gates until their child is 3 to 4 years old and is able to walk up and down the stairs without their support. Choice D is incorrect. This should be done by the time the child is 7 months old. A child who is developing normally will be crawling, reaching, and pulling themselves up on things by the time they are 7 months old, so this is when items on tables should be moved for their safety.

You are providing education on home safety to a group of new parents. Which of the following educational points are important to include? Select all that apply. A. Use stair gates to keep children off the stairs until they are in kindergarten. B. Cover the electrical outlets by the time your infant is 7 months old. C. Place locks on toilet lids by the time your infant is 7 months old. D. Move items on coffee tables to areas that cannot be reached before your child is a year old.

C Choice C is correct. The first thing that you should do is establish the client's trust. Trust is the early stage of the therapeutic nurse-client relationship. After the trust is established, the nurse should encourage, facilitate, and allow the client to ventilate their feelings. This ventilation of feelings is used for and enfolded into the assessment of the client as well as their current psychosocial functioning; this is often used to generate a nursing diagnosis that is specific to the client's needs. Choice A is incorrect. Although the nurse will assess the client and their current psychosocial functioning, this cannot be done until other phases of the nursing process, and the therapeutic nurse-client relationship is established. Choice B is incorrect. A nursing diagnosis is not established until other phases of the nursing process have been started, including the therapeutic nurse-client relationship. Choice D is incorrect. Although it is necessary to encourage, facilitate, and allow the client to ventilate their feelings, this cannot be done until something else is established.

Your adolescent client has been admitted to the adolescent psychiatric mental health unit. What is the first thing that you should do for this client? A. Assess their current psychosocial functioning. B. Generate a nursing diagnosis. C. Establish trust with the client. D. Allow the client to ventilate their feelings.

B Choice B is correct. You would respond to the client's statement of "I do not want to become a druggie" with "The possible complications of unrelieved pain greatly outweigh the risk of addiction which is very low when a person has no prior history of drug abuse" when your client is reluctant to take a necessary dose of narcotic analgesic for severe pain. Responding in this manner allows you to educate the client about a misconception related to pain management in terms of fears of addiction because only a small number, approximately 5% of people, without a prior history of substance abuse that take narcotic analgesics for pain become addicted to them. This response also educates the client about some of the possible complications of unrelieved pain, such as immobility, atelectasis, and infections. Choice A is incorrect. This is an inappropriate response because it is NOT therapeutic, and it is also false. Approximately 5% of people without a prior history of substance abuse that take narcotic analgesics for pain become addicted to them. Choice C is incorrect. This is an inappropriate response because you have failed to educate the client about their fears of addiction by telling them that only approximately 5% of people without a prior history of substance abuse that take narcotic analgesics for pain become addicted to them. Choice D is incorrect. This is an inappropriate response because you have failed to educate the client about their fears of addiction by telling them that only approximately 5% of people without a prior history of substance abuse that take narcotic analgesics for pain become addicted to them.

Your client is reluctant to take a necessary dose of narcotic analgesic for severe pain. The client states, "I do not want to become a druggie." How would you respond to this client's comment? A. "That is ridiculous. Nobody gets addicted to narcotics when they do not have a prior history of drug abuse." B. "The possible complications of unrelieved pain greatly outweigh the risk of addiction which is very low when a person has no prior history of drug abuse." C. "A lot of people prefer to be brave and stick it out so you are not alone." D. "You have a right to refuse any and all treatments, so just do without it."

A Choice A is correct. This client with chronic pancreatitis and gastroparesis is complaining of a migraine headache. Butorphanol is available in the oral form, transnasal form, transdermal, and parenteral form. The doctor has ordered butorphanol orally as needed for pain, but you would call the doctor and suggest transnasal butorphanol because the client has gastroparesis. Choice B is incorrect. Butorphanol is not available for rectal administration. Choice C is incorrect. You would not administer butorphanol orally for pain because this route is contraindicated among clients with gastroparesis. Choice D is incorrect. You cannot administer transdermal butorphanol without a doctor's order. The doctor needs to be called to obtain such an order.

Your client, who has chronic pancreatitis and gastroparesis, is complaining of a migraine headache. The doctor has ordered butorphanol orally as needed for pain. What would you do? A. Call the doctor and suggest transnasal butorphanol because the client has gastroparesis. B. Call the doctor and suggest rectal butorphanol because the client has pancreatitis. C. Administer the butorphanol orally as ordered. D. Administer the butorphanol transdermally for pain.


Kaugnay na mga set ng pag-aaral

CEH Module 17: Hacking Mobile Platform

View Set

Chapter 06 Manufacturing and Service Processes

View Set